Discussion:
Existenz von Umkehrfunktionen bei Verkettung algebraisch unabhängiger Funktionen
(zu alt für eine Antwort)
IV
2018-04-08 13:26:25 UTC
Permalink
Hallo,

könnt Ihr mir bitte wieder helfen?

Es scheint der mathematische Satz unten zu gelten (siehe
https://mathoverflow.net/questions/296676/algebraic-independence-of-the-composition-of-functions
; Beweis folgt später an anderer Stelle).

Ist dann folgende Behauptung wahr?
Seien f1: Z1 Gebiet \subseteq \mathbb{C} \to \mathbb{C} und f2 \: Z2 Gebiet
\subseteq \mathbb{C} \to \mathbb{C} über dem Koeffizientenkörper \mathbb{C}
algebraisch voneinander unabhängige holomorphe Funktionen und A eine über
dem Koeffizientenkörper \mathbb{C} algebraische Funktion mit A: D \subseteq
\mathbb{C}^2, (z1,z2) \mapsto A(z1,z2).
Dann hat die Funktion F: Z Gebiet \subseteq \mathbb{C} \to \mathbb{C}, z
\mapsto A(f1(z),f2(z)) keine holomorphe Umkehrfunktion.

Denn dadurch, daß entsprechend dem Satz unten f1 \circ g und f2 \circ g auch
für die Umkehrfunktion g der Funktion F über dem Koeffizientenkörper
\mathbb{C} algebraisch unabhängig voneinander sind, läßt sich der Ausdruck
A(f1(g(z)),f2(g(z))) nicht zu A1(z), mit A1 eine über dem
Koeffizientenkörper \mathbb{C} algebraische Funktion mit A1: D{A1} \subseteq
\mathbb{C} \to \mathbb{C}, z \mapsto A1(z), vereinfachen, also auch nicht zu
z. (Wie kann man das beweisen?)


Satz:
Seien f1, f2, g holomorphe Funktionen mit
f1: Z1 Gebiet \subseteq \mathbb{C} \to \mathbb{C},
f2: Z2 Gebiet \subseteq \mathbb{C} \to \mathbb{C},
g: Z3 Gebiet \subseteq \mathbb{C} \to \mathbb{C}, g nicht konstant,
und sei Z1 \cap Z2 \cap g(Z3) \neq \emptyset.
Wenn f1 und f2 über dem Koeffizientenkörper \mathbb{C} algebraisch
unabhängig voneinander sind, dann sind die Funktionen F1: Z \to \mathbb{C},
z \mapsto f1(g(z)) und F2: Z \to \mathbb{C}, z \mapsto f2(g(z)) über dem
Koeffizientenkörper \mathbb{C} algebraisch unabhängig voneinander.



Beide Sätze könnten bisher noch unbekannt zu sein. Sie könnten auch für die
Entscheidbarkeit der Auflösbarkeit von Gleichungen eine Rolle spielen. Es
wäre also lohnend, wenn Ihr hier Eure Erfahrung mit einbringen könntet.

Vielen Dank.
IV
2018-04-08 17:26:52 UTC
Permalink
"IV" schrieb im Newsbeitrag news:pad5a1$7la$***@news.albasani.net...
Daraus ergibt sich dann, daß die Lambert-W-Funktion über einem Gebiet keine
holomorphe Funktion sein kann (Ist das so?) und demzufolge keine Elementare
Funktion.
Martin Vaeth
2018-04-08 18:14:10 UTC
Permalink
die Lambert-W-Funktion über einem Gebiet keine holomorphe
Funktion sein kann (Ist das so?)
Dem ist nicht so. Umkehrfunktionen holomorpher Funktionen
sind immer holomorph. Die Lambert-W-Funktion ist die
Umkehrfunktion der holomorphen Funktio z-> z exp z
(auf geeigneten Riemannschen Flächen oder geeigneten
Teilgebieten im Definitions- und Wertebereich) und
daher automatisch holomorph.
und demzufolge keine Elementare Funktion.
demzufolge(?)
Es ist Folklore, dass die Lamber-W-Funktion nicht elementar
ist (wenn man nur log und exp als einzige transzendente
Funktionen bei der Definitoin elementarer Funktionen zulässt).
IV
2018-04-08 20:18:06 UTC
Permalink
Post by Martin Vaeth
und demzufolge keine Elementare Funktion.
demzufolge(?)
Es ist Folklore, dass die Lambert-W-Funktion nicht elementar ist (wenn man
nur log und exp als einzige transzendente Funktionen bei der Definitoin
elementarer Funktionen zulässt).
demzufolge: siehe Duden, Bedeutungsübersicht: demnach, folglich
Keine verbreitete Folklore allerdings ist der Beweis, daß die
Lambert-W-Funktion nicht elementar ist.
Martin Vaeth
2018-04-09 18:12:03 UTC
Permalink
"Martin Vaeth" schrieb im Newsbeitrag
Post by Martin Vaeth
demzufolge(?)
demzufolge: siehe Duden, Bedeutungsübersicht: demnach, folglich
Eben. Mathematisch also "=>". Nachdem sich aber die Prämisse (linke Seite)
als falcsh herausgestellt hat, vorlor der Satz seine Bedeutung.
Post by Martin Vaeth
Es ist Folklore, dass die Lambert-W-Funktion nicht elementar ist (wenn man
nur log und exp als einzige transzendente Funktionen bei der Definitoin
elementarer Funktionen zulässt).
Keine verbreitete Folklore allerdings ist der Beweis, daß die
Lambert-W-Funktion nicht elementar ist.
Stimmt. Genauso wie der Sachverhalt - nicht aber der Beweis -
Folklore ist, dass exp(x^2) keine elementare Stammfunktion hat.

Ich vermute, so ähnlich wie man Letzteres aus den Arbeiten von
Liouville, Rosenlicht, oder dem Risch-Algorithmus folgern kann,
kann man die Nicht-Elementarität der W-Funktion aus der von der
zitierten Arbeit folgern:
Zunächst erhält man, dass - wenn die Funktion elementar wäre -
sie die beschriebene Form haben müsste. Diese Form macht man
als Ansatz und führt sie "irgendwie" zum Widerspruch.
Vermutlich, indem man "geschickt" die Sachverhalte benutzt, die
in der Arbeit über Funktionen dieser Form erhalten wurden.
Einfach ist das vermutlich nicht, aber trotzdem kein weltbewegendes
Ergebnis.
IV
2018-04-09 19:12:52 UTC
Permalink
Post by IV
Post by Martin Vaeth
Es ist Folklore, dass die Lambert-W-Funktion nicht elementar ist (wenn
man nur log und exp als einzige transzendente Funktionen bei der
Definition elementarer Funktionen zulässt).
Keine verbreitete Folklore allerdings ist der Beweis, daß die
Lambert-W-Funktion nicht elementar ist.
Stimmt. Genauso wie der Sachverhalt - nicht aber der Beweis - Folklore
ist, dass exp(x^2) keine elementare Stammfunktion hat.
...
Ich vermute, so ähnlich wie man Letzteres aus den Arbeiten von Liouville,
Rosenlicht, oder dem Risch-Algorithmus folgern kann, kann man die
Nicht-Elementarität der W-Funktion aus der von der zitierten Arbeit
...
Vermutlich, indem man "geschickt" die Sachverhalte benutzt, die in der
Arbeit über Funktionen dieser Form erhalten wurden.
Und meine Vermutung ist, daß dafür nicht spezielle Eigenschaften der
Elementaren Funktionen benötigt werden, sondern ganz allgemeine, daß also
Ritts Satz auf andere Funktionenklassen erweitert werden könnten - so wie
Liouvilles "Integration in endlichen Termen" später von den Elementaren
Funktionen auf Körper von Funktionen (Rosenlicht), z. B. auf die Klasse der
Liouvilleschen Funktionen, erweitert werden konnte.
Liouville hat die Elementaren Funktionen als Komposition endlicher Anzahlen
exp, ln und/oder algebraische Funktionen dargestellt. Man kann sie als
Körper darstellen.
Neben der "Integration in endlichen Termen" haben sich die maßgeblichen
Autoren (u. a. Liouville, Ritt, Rosenlicht, Bronstein, Davenport) auch mit
Entscheidungsmethoden für die "Elementarität" einer gegebenen Funktion bzw.
ihrer Umkehrfunktion befaßt:
Ritt, J. F.: Elementary functions and their inverses. Trans. Amer. Math.
Soc. 27 (1925) (1) 68-90
http://www.ams.org/journals/tran/1925-027-01/S0002-9947-1925-1501299-9/
Rosenlicht, M.: On the explicit solvability of certain transcendental
equations. Publications mathématiques de l'IHÉS 36 (1969) 15-22
https://eudml.org/doc/103891
Bronstein, M.; Corless, R. M.; Davenport, J. H., Jeffrey, D. J.: Algebraic
properties of the Lambert W Function from a result of Rosenlicht and of
Liouville. Integral Transforms and Special Functions 19 (2008) (10) 709-712
http://opus.bath.ac.uk/27004/
Ritt verwendet Liouvilles Methode, die Elementaren Funktionen entsprechend
ihrer Ordnung (Ich interpretiere diese als Schachtelungstiefe.) zu
klassifizieren und zu zeigen, ob oder ob nicht f^-1 \circ f für eine
gegebene Funktion f in eine Funktion der Ordnung 0 (algebraische Funktion
bzw. Identität) transformiert werden kann.
Einfach ist das vermutlich nicht ...
Na, mit der Methode von Liouville oder dem von Ritt bereitgestellten Satz
ist das leicht. Ich vermute, man braucht nur die algebraische Unabhängigkeit
(deren Begriff Ritt nicht verwendet) der Monome zeigen. Mit dem von
Rosenlicht bereitgestellten Satz für Funktionen eines gegebenen Körpers ist
das mitunter etwas schwieriger.
Einfach ist das vermutlich nicht, aber trotzdem kein weltbewegendes
Ergebnis.
Herauszubekommen ob eine gegebene Funktion oder deren Umkehrfunktion eine
Elementare Funktion ist, ist vielleicht nicht so bedeutsam. Von einiger
Bedeutung ist aber, daß sich damit die Lösbarkeit gegebener Gleichungen in
einer gegebenen Funktionenklasse zeigen läßt. Und das könnte schon die Welt
bewegen (hier und da, ein wenig) - könnte ich mir vorstellen. (Nur, Euch
scheint das nicht zu bewegen.)
Martin Vaeth
2018-04-10 07:48:58 UTC
Permalink
Post by IV
Einfach ist das vermutlich nicht ...
Na, mit der Methode von Liouville oder dem von Ritt bereitgestellten Satz
ist das leicht.
Mit der Methode von Liouville weiß man nur, dass die Stammfunktion -
wenn sie elementar ist - eine spezielle Gestalt haben muss. IIRC
kommen für das konkrete Beispiel e^{x^2} nach Liouvilles Theorie
als elementare Stammfunktionen nur endliche Summen der Gestalt
\sum_n f_n(x)e^{g_n(x)} mit gebrochen-rationalen Funktionen
f_n und g_n in Frage. Es bleibt nachzuweisen, dass keine dieser
Funktionen die Funktion e^{x^2} als Ableitung haben kann.
Da ist also noch deutlich etwas zu tun.
(Dieses letzte Problem habe ich irgendwann mal in einem berühmten
Übungsaufgabenbuch gesehen).

Bei Ritt ist dieses Überprüfen konkreter Beispiele weitaus
schwieriger, weil Kompositionen statt Summen involviert sind.
Ich zumindest sehe nach kurzer Überlegung nicht, weshalb die
W-Funktion nicht in der von Ritt beschriebenen Form
dargestellt werden kann (obwohl ich natürlich aus dem
Folklore-Sachverhalt weiß, dass dem so sein muss).
IV
2018-04-10 16:40:37 UTC
Permalink
Post by IV
Einfach ist das vermutlich nicht ...
Na, mit der Methode von Liouville oder dem von Ritt bereitgestellten Satz
ist das leicht.
Mit der Methode von Liouville weiß man nur, dass die Stammfunktion - wenn
sie elementar ist - eine spezielle Gestalt haben muss. IIRC kommen für das
konkrete Beispiel e^{x^2} nach Liouvilles Theorie als elementare
Stammfunktionen nur endliche Summen der Gestalt \sum_n f_n(x)e^{g_n(x)}
mit gebrochen-rationalen Funktionen f_n und g_n in Frage. Es bleibt
nachzuweisen, dass keine dieser Funktionen die Funktion e^{x^2} als
Ableitung haben kann.
Da ist also noch deutlich etwas zu tun.
(Dieses letzte Problem habe ich irgendwann mal in einem berühmten
Übungsaufgabenbuch gesehen).
Bei Ritt ist dieses Überprüfen konkreter Beispiele weitaus schwieriger,
weil Kompositionen statt Summen involviert sind.
Ich zumindest sehe nach kurzer Überlegung nicht, weshalb die W-Funktion
nicht in der von Ritt beschriebenen Form dargestellt werden kann (obwohl
ich natürlich aus dem Folklore-Sachverhalt weiß, dass dem so sein muss).
Für mich geht es nur (Oder erstmal?) darum,
- Ritts Satz mit sein Wesen ausdrückenden treffenderen mathematischen
Fachbegriffen (Definitionsbereich, Stelligkeit, mehrwertige Relation,
mehrstellige algebraische Funktion, algebraische Unabhängigkeit) zu
formulieren (Warum hat Ritt diese Begriffe 1925 nicht verwendet?),
- ihn zu beweisen,
und
- auf andere und allgemeinere Funktionenklassen zu erweitern.
Wie Du sagst, ist Ritts Satz, genau wie Liouvilles Satz, "nur" ein
Struktursatz (IV). Kann man so sagen?
Zu erkennen, ob eine gegebene Funktion in der im Satz geforderten Form
darstellbar ist oder nicht, ist dann ein anderes Problem. Ritt selbst sagt
dazu:
"It remains to develop a method for recognizing whether a given elementary
function can be reduced to the above form for F(z). How to test fairly
simple functions will be evident from the details of our proofs."
Man könnte das so interpretieren als sei diese Aufgabe recht einfach. Ich
(als Nichtmathematiker) verstehe aber noch nicht mal den Beweis selbst,
obwohl er sich eigentlich recht einfach liest.

Wenn man Ritts Satz auf eine gegebene konkrete Funktion anwenden will, muß
man die gegebene Funktion in exp, ln und/oder algebraische Funktionen mit
der Komposition als Verknüpfung zerlegen (Kann man so sagen?), man muß also
eine ihrer Kompositionsdarstellungen (H0Iger Schulz) finden.
Da auch die Identität eine algebraische Funktion ist, ist die äußerste
Funktion in der Kompositionsdarstellung immer eine algebraische Funktion
(IV), bei Liouville und Ritt ist diese Weisheit nur implizit formuliert.
Für F(z) = z * exp(z) ergibt sich: z * exp(z) = A(z, exp(z)), wobei A eine
(zweistellige) algebraische Funktion ist mit A: (z1, z2) |--> z1 * z2. Nach
Ritt muß man nun zeigen, daß es für f keine Kompositionsdarstellung gibt,
deren äußere Funktion eine algebraische Funktion mit weniger als den zwei
Monomen von oben z und exp(z) gibt.
Nachdem mir aufgefallen war, daß Ausdrücke, die sich nicht zu einer
(einstelligen) algebraischen Funktion eines einzigen Monoms, also zu (
Alpha(g(z)) und zu id_z, worin Alpha eine algebraische Funktion und g eine
transzendente Funktion ist) vereinfachen lassen, paarweise voneinander
algebraisch unabhängige Monome haben müssen, fand ich später bei Ritt die
entsprechende Stelle:
"But of all the equations (2) which determine u, there are some which
involve a minimum number of monomials of order n; that is, the r_n in (N +
I) of § 9 is a minimum. In that case, no algebraic relation can exist
between these r_n monomials of order n and monomials of order less than n.
We mean by this that ..."
Offenbar war meine Vermutung richtig. Und auch der von mir formulierte Satz,
wo die forschenden Mathematiker in MathOverflow meinten, es sei
offensichtlich, daß der Satz für holomorphe nirgendwo (IV) konstante
Funktionen gelten muß.
Ich möchte damit dann die bei Ritt noch zwischen seinen Beweiszeilen
versteckt liegende Methode ersetzen durch die Aufgabe, die algebraische
Unabhängigkeit aller Monome der äußeren algebraischen Funktion in der
Kompositionsdarstellung der zu untersuchenden Funktion voneinander zu
zeigen. Zumindest bei wenigverketteten (wenigverschachtelten) Monomen ist
das einfach.
Ich zumindest sehe nach kurzer Überlegung nicht, weshalb die W-Funktion
nicht in der von Ritt beschriebenen Form dargestellt werden kann (obwohl
ich natürlich aus dem Folklore-Sachverhalt weiß, dass dem so sein muss).
Meine Beobachtung: exp und ln sind einstellig, algebraische Funktionen 1-
bis n-stellig. In der in Ritts Satz für elementare Funktionen mit
elementarer Umkehrfunktion geforderten Kompositionsdarstellung treten keine
mehrstelligen algebraischen Funktionen auf. (Das ist das was ich an anderer
Stelle mit (geometrisch) "linear" bezeichnet hatte.)
A(z, exp(z)) läßt sich nicht vereinfachen zu einer einstelligen
algebraischen Funktion nur eines einzigen Monoms. Ich vermute, der Grund
dafür liegt bei unverschachtelten Monomen wie hier einfach darin, daß die
Monome über dem Koeffizientenkörper \mathbb{C} algebraisch unabhängig sind.
Martin Vaeth
2018-04-10 20:04:11 UTC
Permalink
Post by IV
"It remains to develop a method for recognizing whether a given elementary
function can be reduced to the above form for F(z). How to test fairly
simple functions will be evident from the details of our proofs."
Deswegen hatte ich in einem früheren Posting auch vermutet,
dass das für die W-Funktion mit Methoden aus Ritts Arbeit geht.
Ob dem so ist, hatte ich natürlich nicht versucht, zu verifizieren.

Deine anderen Fragen (eine genaue Ausarbeitung der Details von
Ritts Artikel) sind bzgl. Umfang und Schwierigkeit eine Bachelor-
oder Masterarbeit.
IV
2018-04-10 20:34:18 UTC
Permalink
Deine anderen Fragen (eine genaue Ausarbeitung der Details von Ritts
Artikel) sind bzgl. Umfang und Schwierigkeit eine Bachelor- oder
Masterarbeit.
Genau meine Meinung. Nur, es findet sich niemand. Es wäre eine relativ
einfache aber lohnende Aufgabe. (Man kann in die Geschichte und in die
Lehrbücher kommen.)
Für einen angehenden Mathematiker sind die Zusammenhänge und benötigten
Kenntnisse eigentlich relativ einfach.
Ich bin ja eigentlich nur daran interessiert, daß das (für die Menschheit)
gemacht wird - wenn durch jemand Anderen, umso besser, dann kann ich, der
ich doch kein Mathematiker bin, endlich mit der Plackerei aufhören und mich
für mich weniger aufwendigen Themen zuwenden.
IV
2018-04-11 15:11:56 UTC
Permalink
Deine anderen Fragen (eine genaue Ausarbeitung der Details von Ritts
Artikel) sind bzgl. Umfang und Schwierigkeit eine Bachelor- oder
Masterarbeit.
Genau mein Reden.
Die Arbeit würde sich lohnen - nicht nur für die Menschheit, man käme sogar
in die Lehr- und Geschichtsbücher.
Aber welche Profs wollen sich mit sowas "Simplem" beschäftigen? Ich kann ja
schlecht alle Funktionstheoretiker anschreiben.
Ein Mathematiker wäre viel geeigneter als ich - für mich ist das eine
aufwendige, zeitraubende Plackerei. Wenn sich jemand Kompetentes findet
könnte ich mich wieder meinen anderen Themen widmen.
H0Iger SchuIz
2018-04-11 15:35:03 UTC
Permalink
Post by IV
Deine anderen Fragen (eine genaue Ausarbeitung der Details von Ritts
Artikel) sind bzgl. Umfang und Schwierigkeit eine Bachelor- oder
Masterarbeit.
Genau mein Reden.
Die Arbeit würde sich lohnen - nicht nur für die Menschheit, man käme sogar
in die Lehr- und Geschichtsbücher.
Hört, hört. Allerdings frage ich mich, welche hellseerischen Fähigkiten
das bedründen sollen. Das Niveau "Bachelor- oder Masterarbeit" sichert
mit Nichten einen Platz in den Annalen der Mathematik. Solche werden
jährlich zu Tausenden verfasst und außer von den Gutachtern nicht
gelesen.
Post by IV
Aber welche Profs wollen sich mit sowas "Simplem" beschäftigen?
Simpel und geschichtträchtig? Da lecjte sich doch jeder die Finger nach.

Ganz ehrlich? Lnagsam wird's lächerlich.

hs
IV
2018-04-11 20:50:43 UTC
Permalink
Post by IV
Deine anderen Fragen (eine genaue Ausarbeitung der Details von Ritts
Artikel)
Genau mein Reden.
Die Arbeit würde sich lohnen - nicht nur für die Menschheit, man käme
sogar in die Lehr- und Geschichtsbücher.
Hört, hört. Allerdings frage ich mich, welche hellseherischen Fähigkeiten
das begründen sollen.
Post by IV
Aber welche Profs wollen sich mit sowas "Simplem" beschäftigen?
Simpel und geschichtsträchtig? Da leckte sich doch jeder die Finger nach.
Ganz ehrlich? Langsam wird's lächerlich.
Zum Lachen:
Ein Struktursatz der sagt welche bijektiven Funktionen welcher
Funktionenklassen globale oder lokale Umkehrfunktionen in welchen
Funktionenklassen haben können und welche nicht, wäre nicht ganz
unbedeutend.
Ein Struktursatz der sagt welche Nullstellengleichungen von Funktionen
welcher Funktionenklassen durch algebraische Umformung lösbar sind und
welche nicht, wäre nicht ganz unbedeutend.
Alle Mathematiker, so auch die hier anwesenden, "can easily see" diese
mathematischen Zusammenhänge und Sätze
(siehe
https://mathoverflow.net/questions/296676/algebraic-independence-of-the-composition-of-functions
),
tausenden mathematischen Laien aber wären diese Sätze eine Offenbarung und
Arbeitserleichterung. Das weiß man auch ohne hellseherische Fähigkeiten.
H0Iger SchuIz
2018-04-13 06:51:55 UTC
Permalink
Post by IV
tausenden mathematischen Laien aber wären diese Sätze eine Offenbarung
Ah, jetzt sind wir schon bei Offenbarungen.

hs
Christian Gollwitzer
2018-04-11 21:27:38 UTC
Permalink
Post by IV
Ein Mathematiker wäre viel geeigneter als ich - für mich ist das eine
aufwendige, zeitraubende Plackerei.
Und Du denkst, der Mathematiker schüttelt das einach aus dem Ärmel? Da
muss ich Dich enttäuschen, der arbeitet dann eben auch genauso hart
dafür. Das ist ja auch mit der Grund, dass hier in der Gruppe keiner
"mal schnell" einen Beweis für Deine Behauptungen aufschreibt.
Post by IV
Wenn sich jemand Kompetentes findet
könnte ich mich wieder meinen anderen Themen widmen.
Das kannst Du doch auch so! Warum muss unbedingt erst dieses Thema
abgearbeitet sein? Wer verlangt diese Beweise von Dir? In der Forschung
gewinnt nicht immer der, der sich festbeißt, sondern vielmehr der, der
einen Holzweg verlässt und woanders grüneres Gras findet.

Christian
IV
2018-04-12 16:59:08 UTC
Permalink
Post by IV
Ein Mathematiker wäre viel geeigneter als ich - für mich ist das eine
aufwendige, zeitraubende Plackerei. Wenn sich jemand Kompetentes findet
könnte ich mich wieder meinen anderen Themen widmen.
Und Du denkst, der Mathematiker schüttelt das einfach aus dem Ärmel? Da
muss ich Dich enttäuschen, ...
Na, aber manche Dinge schon - siehe meinen Link auf den Kommentar mit "one
can easily see" bei MathOverflow.
Post by IV
Wenn sich jemand Kompetentes findet könnte ich mich wieder meinen anderen
Themen widmen.
Das kannst Du doch auch so! Warum muss unbedingt erst dieses Thema
abgearbeitet sein? Wer verlangt diese Beweise von Dir?
Weil dieses Thema Potential hat - auf Verallgemeinerung, auf
Abarbeitbarkeit, auf Bedeutsamkeit. Es ist zu schade, um weiterhin nicht
beachtet zu werden.
In der Forschung gewinnt nicht immer der, der sich festbeißt, sondern
vielmehr der, der einen Holzweg verlässt und woanders grüneres Gras
findet.
Man kann das so sehen oder so.
(Mit meiner langjährigen Erfahrung bei der Ideenfindung, Planung,
Organisation, Durchführung und Abrechnung einer Vielzahl vielfältigster
Entwicklungsprojekte über mehrere Fachgebiete hinweg, u. a. in
Naturwissenschaften, naturwissenschaftlicher Informatik, Informatik und
Mathematik, glaube ich ein Gespür dafür entwickelt zu haben, welche Themen
und Strategien Potential haben.)
Christian Gollwitzer
2018-04-12 18:23:29 UTC
Permalink
"Christian Gollwitzer"  schrieb im Newsbeitrag
In der Forschung gewinnt nicht immer der, der sich festbeißt, sondern
vielmehr der, der einen Holzweg verlässt und woanders grüneres Gras
findet.
Man kann das so sehen oder so.
(Mit meiner langjährigen Erfahrung bei der Ideenfindung, Planung,
Organisation, Durchführung und Abrechnung einer Vielzahl vielfältigster
Entwicklungsprojekte über mehrere Fachgebiete hinweg, u. a. in
Naturwissenschaften, naturwissenschaftlicher Informatik, Informatik und
Mathematik, glaube ich ein Gespür dafür entwickelt zu haben, welche
Themen und Strategien Potential haben.
Und ich glaube, dass Du da im vorliegenden Falle falsch liegst. Mein
Gefühl ist - ausgehend von den Beiträgen hier zu Deinen Themen - dass
das keine Sau interessiert. Sorry, das so hart zu sagen, aber wenn das
so interessant wäre, würdest Du bessere Antworten bekommen bzw. auch
mehr Literatur dazu finden.

Christian
IV
2018-04-12 20:13:54 UTC
Permalink
Post by Christian Gollwitzer
Und ich glaube, dass Du da im vorliegenden Falle falsch liegst. Mein
Gefühl ist - ausgehend von den Beiträgen hier zu Deinen Themen - dass das
keine Sau interessiert. Sorry, das so hart zu sagen, aber wenn das so
interessant wäre, würdest Du bessere Antworten bekommen bzw. auch mehr
Literatur dazu finden.
Tja, irren ist menschlich.
Martin Vaeth
2018-04-13 06:23:48 UTC
Permalink
dass das keine Sau interessiert.
Um das etwas auszuführen: Es gibt sehr wenige Mathematiker
(und noch viel weniger Stellen, auf denen man tatsächlich frei
Mathematik machen kann) und eine riesige Menge interessanter
Probleme in der Mathematik. Außerdem noch eine riesige Menge
an Problemen, die zwar mathematisch manchmal nicht so spannend
sind, aber für verschiedene Anwendungen extrem wichtig.
Jeder Mathematiker muss sich sehr gut überlegen, für welche
Fragen er Zeit investiert. Und Studenten will man natürlich
bei der Vergabe von Themen erst recht nicht in eine berufliche
Sackgasse führen.
Vor diesem traurigen Hintergrund ist es ganz natürlich, dass
sich Modeströmungen bilden. Die Computer-Algebra, zu der die
benannte Frage wohl gehört, interessiert sich derzeit wohl
für ganz andere Fragen. Ich vermute, automatische Beweiser
und Verifizierer sind dort derzeit aktuell.
IV
2018-04-13 16:04:26 UTC
Permalink
Post by IV
Aber welche Profs wollen sich mit sowas "Simplem" beschäftigen? Ich kann
ja schlecht alle Funktionstheoretiker anschreiben.
Ein Mathematiker wäre viel geeigneter als ich ...
Wenn sich jemand Kompetentes findet ...
Um das etwas auszuführen: Es gibt sehr wenige Mathematiker (und noch viel
weniger Stellen, auf denen man tatsächlich frei Mathematik machen kann)
und eine riesige Menge interessanter Probleme in der Mathematik. Außerdem
noch eine riesige Menge an Problemen, die zwar mathematisch manchmal nicht
so spannend sind, aber für verschiedene Anwendungen extrem wichtig.
Jeder Mathematiker muss sich sehr gut überlegen, für welche Fragen er Zeit
investiert. Und Studenten will man natürlich bei der Vergabe von Themen
erst recht nicht in eine berufliche Sackgasse führen.
Vor diesem traurigen Hintergrund ist es ganz natürlich, dass sich
Modeströmungen bilden. Die Computer-Algebra, zu der die benannte Frage
wohl gehört, interessiert sich derzeit wohl für ganz andere Fragen. Ich
vermute, automatische Beweiser und Verifizierer sind dort derzeit aktuell.
Aber das weiß ich doch.
"In der Forschung gewinnt nicht immer der, der sich festbeißt, sondern
vielmehr der, der einen Holzweg verlässt und woanders grüneres Gras findet."
Es soll aber auch Mathematiker geben, die etwas Freizeit haben.
H0Iger SchuIz
2018-04-15 10:27:07 UTC
Permalink
Post by IV
Es soll aber auch Mathematiker geben, die etwas Freizeit haben.
Ja? Und?

hs
IV
2018-04-08 22:28:39 UTC
Permalink
Post by IV
Daraus ergibt sich dann, daß die Lambert-W-Funktion über einem Gebiet
keine holomorphe Funktion sein kann (Ist das so?)
Dem ist nicht so. Umkehrfunktionen holomorpher Funktionen sind immer
holomorph. Die Lambert-W-Funktion ist die Umkehrfunktion der holomorphen
Funktio z-> z exp z (auf geeigneten Riemannschen Flächen oder geeigneten
Teilgebieten im Definitions- und Wertebereich) und daher automatisch
holomorph.
W: Lambert-W-Funktion
Sind W und e^W algebraisch unabhängig voneinander über \mathbb{C} über jedem
Gebiet das Teilmenge des Durchschnitts der Definitionsbereiche beider
Funktionen ist? Ich denke mal ja. Dann ist vielleicht der von mir genannte
Satz wahr, nicht aber meine Behauptung ganz oben.
Eine Funktion z \mapsto A(f1(g(z)),f2(g(z))) worin (f1 \circ g) und (f_2
\circ g) algebraisch unabhängig voneinander über \math{C} sind kann dann
offenbar irgendwie anders zu einer Funktion z \mapsto A1(z) und zu z \mapsto
z (id_z) vereinfacht werden. Aber wie?
IV
2018-04-10 20:25:45 UTC
Permalink
die Lambert-W-Funktion über einem Gebiet keine holomorphe Funktion sein
kann (Ist das so?)
Dem ist nicht so. Umkehrfunktionen holomorpher Funktionen sind immer
holomorph. Die Lambert-W-Funktion ist die Umkehrfunktion der holomorphen
Funktio z-> z exp z (auf geeigneten Riemannschen Flächen oder geeigneten
Teilgebieten im Definitions- und Wertebereich) und daher automatisch
holomorph.
F(z) = z * exp(z)
F(z) = A(z, exp(z)), mit A: eine zweistellige algebraische Funktion
Die Funktionen id_z und exp sind über dem Koeffizientenkörper \mathbb{C}
algebraisch unabhängig. Da (warum auch immer) keine Darstellung F(z) =
A1(theta(z)), mit A eine einstellige algebraische Funktion und theta eine
transzendente elementare Funktion (nach Liouville und Ritt: ein Monom
gewissser Ordnung) existiert, kann nach Ritts Satz in [Ritt 1925] die
Umkehrfunktion Phi von F, wenn sie existiert, keine elementare Funktion
sein. Meine These ist, daß zur Darstellung von Phi als elementare Funktion
die Umkehrfunktion von A benötigt wird, die Inverse aber eine mehrstellige
Relation ist, zu A also keine Umkehrfunktion existiert.
Nach meinem Satz in dieser Diskussion ganz oben ist auch A(Phi(z),
exp(Phi(z)), wenn alle Funktionen holomorph und nirgendwo konstant sind,
eine zweistellige algebraische Funktion über dem Koeffizientenkörper
\mathbb{C} algebraisch unabhängiger Monome. Was führt nun dazu, daß die
Lambert-W-Funktion Phi den Ausdruck vereinfacht? Ist es lokale
Nicht-Holomorphie, ein lokal nicht zusammenhängender Definitionsbereich,
oder was?:
Phi = W, W ist LambertW
z * exp(z) = A(z, exp(z)) : alle Monome sind algebraisch unabhängig
W(z) * exp(W(z)) = A1(W(z), exp(W(z))): alle Monome sind algebraisch
unabhängig
W(z) * exp(W(z)) = z => exp(W(z)) = z / W(z)
A1(W(z), z/W(z)): alle Monome sind algebraisch unabhängig
Da nun A1: (z1, z2) |-> z1 * z2, vereinfacht sich der Ausdruck zu z - und
zwar obwohl W und z/W algebraisch unabhängig sind.
Woran liegt das?
Torn Rumero DeBrak
2018-04-08 18:36:53 UTC
Permalink
Post by IV
Hallo,
könnt Ihr mir bitte wieder helfen?
Es scheint der mathematische Satz unten zu gelten (siehe
https://mathoverflow.net/questions/296676/algebraic-independence-of-the-composition-of-functions
; Beweis folgt später an anderer Stelle).
Ist dann folgende Behauptung wahr?
Seien f1: Z1 Gebiet \subseteq \mathbb{C} \to \mathbb{C} und f2 \: Z2
Gebiet \subseteq \mathbb{C} \to \mathbb{C} über dem Koeffizientenkörper
\mathbb{C} algebraisch voneinander unabhängige holomorphe Funktionen und
A eine über dem Koeffizientenkörper \mathbb{C} algebraische Funktion mit
A: D \subseteq \mathbb{C}^2, (z1,z2) \mapsto A(z1,z2).
Dann hat die Funktion F: Z Gebiet \subseteq \mathbb{C} \to \mathbb{C}, z
\mapsto A(f1(z),f2(z)) keine holomorphe Umkehrfunktion.
Denn dadurch, daß entsprechend dem Satz unten f1 \circ g und f2 \circ g
auch für die Umkehrfunktion g der Funktion F über dem
Koeffizientenkörper \mathbb{C} algebraisch unabhängig voneinander sind,
läßt sich der Ausdruck A(f1(g(z)),f2(g(z))) nicht zu A1(z), mit A1 eine
D{A1} \subseteq \mathbb{C} \to \mathbb{C}, z \mapsto A1(z),
vereinfachen, also auch nicht zu z. (Wie kann man das beweisen?)
Seien f1, f2, g holomorphe Funktionen mit
f1: Z1 Gebiet \subseteq \mathbb{C} \to \mathbb{C},
f2: Z2 Gebiet \subseteq \mathbb{C} \to \mathbb{C},
g: Z3 Gebiet \subseteq \mathbb{C} \to \mathbb{C}, g nicht konstant,
und sei Z1 \cap Z2 \cap g(Z3) \neq \emptyset.
Wenn f1 und f2 über dem Koeffizientenkörper \mathbb{C} algebraisch
unabhängig voneinander sind, dann sind die Funktionen F1: Z \to
\mathbb{C}, z \mapsto f1(g(z)) und F2: Z \to \mathbb{C}, z \mapsto
f2(g(z)) über dem Koeffizientenkörper \mathbb{C} algebraisch unabhängig
voneinander.
Beide Sätze könnten bisher noch unbekannt zu sein. Sie könnten auch für
die Entscheidbarkeit der Auflösbarkeit von Gleichungen eine Rolle
spielen. Es wäre also lohnend, wenn Ihr hier Eure Erfahrung mit
einbringen könntet.
Vielen Dank.
Also sind wir jetzt bei holomorphen Funktionen angekommen, einer Klasse
von Funktionen, die fast alle bekannten "schönen" Eigenschaften besitzen
(wie unendlich oft stetig differenzierbar oder in eine konvergente
Potenzreihe entwickelbar zu sein).

Was mir aber fehlt, ist deine Definition für "über \mathbb{C}
algebraisch abhängige oder unabhängige Funktionen", die man auch
nachprüfen kann, d.h. keine nur rein textuelle Definition,
die wieder auf andere, in der Algebra oder woanders definierte,
Ausdrücke Bezug nimmt.

z.B. eine Definition in der Art:

Zwei holomorphe Funktionen f1: G \subseteq \mathbb{C} \to \mathbb{C}
und f2: G \subseteq \mathbb{C} \to \mathbb{C} heißen über \mathbb{C}
algebraisch abhängig (beachte, daß beide Funktionen denselben
Definitionsbereich besitzen, denn sonst kannst du sie ja nicht überall
vergleichen!), wenn es ein Polynom
P(X,Y)=\(\sum \limits_{i=1}^n \(\sum \limits_{j=1}^m \(a_ij\)*X^i*Y^j \)\)
mit Koeffizienten \(a_ij\) aus \mathbb{C}, die nicht alle gleich Null
sind, gibt, so daß P(f1, f2) = 0 .

Übrigens, hast du eine Vorstellung, was dann genau P(f1, f2) = 0
bedeutet?
Ich frage ja nur, da du immer betonst, kein Mathematiker zu sein.
Sind dann z.B. die Funktionen
f1(z) = exp(z) und f2(z) = ln(z+1)+1 (mit passenden Definitionsbereich)
algebraisch abhängig, da ja für das Polynom P(X, Y) = X - Y
P(f1, f2) = exp(z) - ln(z+1) - 1 für z=0 den Wert 0 besitzt?
Meine Frage läuft darauf hinaus, ob du weist, wann zwei Funktionen
gleich sind.

Aloha
IV
2018-04-08 21:04:55 UTC
Permalink
Post by Torn Rumero DeBrak
Also sind wir jetzt bei holomorphen Funktionen angekommen, einer Klasse
von Funktionen, die fast alle bekannten "schönen" Eigenschaften besitzen
(wie unendlich oft stetig differenzierbar oder in eine konvergente
Potenzreihe entwickelbar zu sein).
Und unendlich oft komplex differnzierbar in einer Umgebung jedes Punktes
ihres offenen Definitionsbereiches.
Die Elementaren Funktionen sind fast überall analytisch und fast überall
stetig. Sie sind meromorph.
Post by Torn Rumero DeBrak
Meine Frage läuft darauf hinaus ob du weißt, wann zwei Funktionen gleich
sind.
Ja, ich weiß es.
Post by Torn Rumero DeBrak
Was mir aber fehlt, ist deine Definition für "über \mathbb{C} algebraisch
abhängige oder unabhängige Funktionen", die man auch nachprüfen kann, d.h.
keine nur rein textuelle Definition, die wieder auf andere, in der Algebra
oder woanders definierte, Ausdrücke Bezug nimmt.
So ist halt Mathematik.
Wir wollen doch korrekt bleiben: ü b e r d e m K o e f f i z i e n t e n k
ö r p e r \mathbb{C} algebraisch voneiander abhängige oder unabhängige
Funktionen.
Die Definition algebraisch abhängiger oder unabhängiger Funktionen ist
Folklore.
Weil ich nichts Anderes Passendes gefunden habe, habe ich mir diese
Definition hier überlegt:
Die Funktionen f1 mit f1: Z1 --> Y1 und f2 mit f2: Z2 --> Y2 sind genau dann
algebraisch unabhängig voneinander über dem Koeffizientenkörper \mathbb{K},
wenn \exists za \in Z1 \cap Z2 \neq \emptyset: f2(za)=A(f1(za)) und \exists
zb \in Z1 \cap Z2 \neq \emptyset: f2(zb) \neq A(f1(zb)), mit A: eine
algebraische Funktion über dem Koeffizientenkörper \mathbb{K}.
H0Iger SchuIz
2018-04-09 11:08:00 UTC
Permalink
Post by IV
So ist halt Mathematik.
Das ist ja schön, dass man das hier mal erklärt bekommt.
Post by IV
Die Definition algebraisch abhängiger oder unabhängiger Funktionen ist
Folklore.
Weil ich nichts Anderes Passendes gefunden habe
Ganz so folkloristisch scheint's dann doch nicht zu sein.
Ich habe jetzt nicht jedes Detail von Jürgens bisherigen Posts parat,
aber war es nicht so, dass er bekannte Sätze verwenden wollte, die die
algebraische (Un)-Abhängigkeit bereits verwenden? Dann müsste da ja auch
irgendwo eine Definition zu finden sein. Eine jetzt neu "überlegte"
Definition ist womöglich zu der in diesen Sätzen vorausgesetzen nicht
äquivalant. Da kann man sich ordentlich ins Knie schießen.

hs
IV
2018-04-09 16:12:34 UTC
Permalink
Post by H0Iger SchuIz
Post by IV
Post by Torn Rumero DeBrak
Was mir aber fehlt, ist deine Definition für "über \mathbb{C}
algebraisch abhängige oder unabhängige Funktionen", die man auch
nachprüfen kann, d.h. keine nur rein textuelle Definition, die wieder
auf andere, in der Algebra oder woanders definierte, Ausdrücke Bezug
nimmt.
Die Definition algebraisch abhängiger oder unabhängiger Funktionen ist
Folklore.
...
Ganz so folkloristisch scheint's dann doch nicht zu sein.
aber war es nicht so, dass er bekannte Sätze verwenden wollte, die die
algebraische (Un)-Abhängigkeit bereits verwenden? Dann müsste da ja auch
irgendwo eine Definition zu finden sein.
Dem ist wohl nicht so. Ich für meinen Teil hatte in Ritts Artikel erkannt,
daß dort der Begriff "Algebraische Unabhängigkeit" treffend ist und verwende
ihn deshalb.
Später dann habe ich in Ritt, J. F.: Elementary functions and their
inverses. Trans. Amer. Math. Soc. 27 (1925) (1) 68-90 gefunden:
"But of all the equations (2) which determine u, there are some which
involve a minimum number of monomials of order n; that is, the r_n in (N+I)
of § 9 is a minimum. In that case, no algebraic relation can exist between
these r_n monomials of order to and monomials of order less than n."
Post by H0Iger SchuIz
Eine jetzt neu "überlegte" Definition ist womöglich zu der in diesen
Sätzen vorausgesetzen nicht äquivalant. Da kann man sich ordentlich ins
Knie schießen.
Ich kenne verschiedene Definitionen algebraisch unabhängiger Zahlen und
Elemente, z. B. aus Artin: Algebra.
Mit den Definitionen algebraischer Unabhängigkeit von Funktionen in
folgenden Artikeln, wenn es dort die Definitionen gibt, kann ich im Moment
aber noch gar nichts anfangen:
Algebraic independence of certain arithmetic functions, 1948
Algebraic independence of elementary functions and its application to
Masser's vanishing theorem, 1990
On the algebraic independence of functions of a certain class, 2013
Da ich noch nicht körpertheoretisch arbeite (z. B. Definition algebraischer
Unabhängigkeit in van der Waerden: Algebra I) und demzufolge noch nicht weiß
ob man damit weiterkommt, zeige ich die Existenz oder Nichtexistenz einer
algebraischen Relation zwischen den Funktionen über die Existenz voneinander
algebraisch unabhängiger Funktionswerte, für die es ja zumindest einige
wenige Transzendenzsätze gibt.
In
https://mathoverflow.net/questions/296676/algebraic-independence-of-the-composition-of-functions
haben die richtigen Mathematiker geschrieben: "one can easily see ...". Die
Antwort selber zu finden kostet mich aber Tage und Wochen. Mit meiner
Definition oben oben bin ich auch auf die nichtkonstanten Holomorphen
Funktionen über zusammenhängendem Definitionsbereich gekommen.
Ihr seid doch solche Koryphäen. Wie würdet Ihr denn die algebraische
Unabhängigkeit der Funktionen exp und ln über dem Koeffizientenkörper
\mathbb{C} beweisen?
(Mir scheint aber eher, daß die größten Kritiker hier doch nicht so fit sind
in algebraischen und transzendenten Funktionenkörpern und algebraischer
Unabhängigkeit von Funktionen im allgemeinen Fall. Ist auch egal - laßt es
uns gemeinsam versuchen.)
Detlef Müller
2018-04-09 16:56:36 UTC
Permalink
"H0Iger SchuIz"  schrieb im Newsbeitrag
[...]
Ihr seid doch solche Koryphäen. Wie würdet Ihr denn die algebraische
Unabhängigkeit der Funktionen exp und ln über dem Koeffizientenkörper
\mathbb{C} beweisen?
Mit Torns Definition für algebraische Abhängigkeit geht jedenfalls
wohl ein klassischer Widerspruchsbeweis:
Angenommen, es gibt ein nicht konstantes Polynom F in IC[u,v],
etwa F = Summe(a_{ij}u^iv^j) mit (1)
F(exp(z),ln(z)) = 0 (Nullpolynom),
so gibt es ein Monom a_{kl}u^kv^l, wobei
a_{kl}\neq0 und (k,l) lexikografisch maximal ist (also k
maximal und mit diesem k dann l maximal).
Für die Rationale Funktion H(u,v):=F/(u^kv^l) konvergiert dann
H(exp(x),ln(x)): IR_+ --> IC
für x gegen Unendlich gegen a_{kl} (was sich aus dem Wachstumsverhalten
von exp und ln in IR ergibt).
Da aber H(exp(x),ln(x)) wegen (1) konstant 0 ist,
folgt a_{kl}=0 im Widerspruch zur Wahl von a_{kl}.

Gruß,
Detlef
--
Dr. Detlef Müller,
http://www.mathe-doktor.de oder http://mathe-doktor.de
IV
2018-04-09 19:24:16 UTC
Permalink
Wie würdet Ihr denn die algebraische Unabhängigkeit der Funktionen exp
und ln über dem Koeffizientenkörper \mathbb{C} beweisen?
Mit Torns Definition für algebraische Abhängigkeit geht jedenfalls wohl
Angenommen, es gibt ein nicht konstantes Polynom F in IC[u,v], etwa F =
Summe(a_{ij}u^iv^j) mit (1) F(exp(z),ln(z)) = 0 (Nullpolynom), so gibt es
ein Monom a_{kl}u^kv^l, wobei a_{kl}\neq0 und (k,l) lexikografisch maximal
ist (also k maximal und mit diesem k dann l maximal).
Für die Rationale Funktion H(u,v):=F/(u^kv^l) konvergiert dann
H(exp(x),ln(x)): IR_+ --> IC für x gegen Unendlich gegen a_{kl} (was sich
aus dem Wachstumsverhalten von exp und ln in IR ergibt).
Da aber H(exp(x),ln(x)) wegen (1) konstant 0 ist, folgt a_{kl}=0 im
Widerspruch zur Wahl von a_{kl}.
Ah, ja, hast Du eine Literaturstelle oder Schlagwörter für mich?

Und ist meine Definition auch richtig? Die ist für mich (jedenfalls im
Moment noch) wesentlich anschaulicher.
Detlef Müller
2018-04-10 10:05:04 UTC
Permalink
Post by IV
Post by Detlef Müller
Wie würdet Ihr denn die algebraische Unabhängigkeit der Funktionen
exp und ln über dem Koeffizientenkörper \mathbb{C} beweisen?
Mit Torns Definition für algebraische Abhängigkeit geht jedenfalls
Angenommen, es gibt ein nicht konstantes Polynom F in IC[u,v], etwa F
= Summe(a_{ij}u^iv^j) mit (1) F(exp(z),ln(z)) = 0 (Nullpolynom), so
gibt es ein Monom a_{kl}u^kv^l, wobei a_{kl}\neq0 und (k,l)
lexikografisch maximal ist (also k maximal und mit diesem k dann l
maximal).
Für die Rationale Funktion H(u,v):=F/(u^kv^l) konvergiert dann
H(exp(x),ln(x)): IR_+ --> IC für x gegen Unendlich gegen a_{kl} (was
sich aus dem Wachstumsverhalten von exp und ln in IR ergibt).
Da aber H(exp(x),ln(x)) wegen (1) konstant 0 ist, folgt a_{kl}=0 im
Widerspruch zur Wahl von a_{kl}.
Ah, ja, hast Du eine Literaturstelle oder Schlagwörter für mich?
Nein, da wurde nur etwas Grenzwertrechnung benutzt ...
wenn der Beweis so geht (man sollte ihn anderen zur Kontrolle vorlegen,
Stichwort "peer-rewiew"), könnte man überlegen, ob die Methode
hilft, z.B. auch ln(x) und ln(x+1) auf algebraische Unabhängigkeit zu
untersuchen, indem man sich auf verschiedenen Pfaden den Singularitäten
(0 bzw. -1) nähert.
Post by IV
Und ist meine Definition auch richtig? Die ist für mich (jedenfalls im
Moment noch) wesentlich anschaulicher.
Wenn ich das richtig sehe, ist deine Definition, daß f1, f2 algebraisch
abhängig sind, wenn es eine algebraische Funktion A mit
f2(z)=A(f1(z)) gibt. Mit dem ganzen sich ergebenden etwas lästigen
"Gefruckel" mit den Definitionsbereichen ...
A soll algebraisch sein, daher muß es ein irreduzibles Polynom P in
IC[X,Y] geben mit P(A(x),x) konstant 0.

Dann wäre P(A(f1(z)),f1(z)) gleich P(f2(z),f1(z)) konstant 0.
f1 und f2 sind dann also algebraisch abhängig.

Die Andere Richtung könnte haariger zu zeigen sein ... vielleicht
mit dem Satz über implizite Funktionen?
Womöglich müssen dann gewisse Punkte aus dem Defbereich genommen
werden.

Gruß,
Detlef
--
Dr. Detlef Müller,
http://www.mathe-doktor.de oder http://mathe-doktor.de
IV
2018-04-10 16:53:20 UTC
Permalink
Post by Detlef Müller
Post by IV
Wie würdet Ihr denn die algebraische Unabhängigkeit der Funktionen exp
und ln über dem Koeffizientenkörper \mathbb{C} beweisen?
Mit Torns Definition für algebraische Abhängigkeit geht jedenfalls wohl
ein klassischer Widerspruchsbeweis: ...
Und ist meine Definition auch richtig? Die ist für mich (jedenfalls im
Moment noch) wesentlich anschaulicher.
Wenn ich das richtig sehe, ist deine Definition, daß f1, f2 algebraisch
abhängig sind, wenn es eine algebraische Funktion A mit f2(z)=A(f1(z))
gibt. Mit dem ganzen sich ergebenden etwas lästigen "Gefruckel" mit den
Definitionsbereichen ...
Ich sehe das folgendermaßen. Ich will doch allgemeine Aussagen für
Funktionenklassen herleiten. Da kann ich doch nicht das Wachstumsverhalten
jeder einzelnen Funktion betrachten. Deshalb mußte ich die algebraisch
Abhängigkeit der Funktionsterme mit der der Funktionswerte in Zusammenhang
bringen.
Torn Rumero DeBrak
2018-04-10 21:35:38 UTC
Permalink
Post by IV
Post by Detlef Müller
Post by IV
Post by Detlef Müller
Wie würdet Ihr denn die algebraische Unabhängigkeit der Funktionen
exp und ln über dem Koeffizientenkörper \mathbb{C} beweisen?
Mit Torns Definition für algebraische Abhängigkeit geht jedenfalls
wohl ein klassischer Widerspruchsbeweis: ...
Und ist meine Definition auch richtig? Die ist für mich (jedenfalls
im Moment noch) wesentlich anschaulicher.
Wenn ich das richtig sehe, ist deine Definition, daß f1, f2
algebraisch abhängig sind, wenn es eine algebraische Funktion A mit
f2(z)=A(f1(z)) gibt. Mit dem ganzen sich ergebenden etwas lästigen
"Gefruckel" mit den Definitionsbereichen ...
Ich sehe das folgendermaßen. Ich will doch allgemeine Aussagen für
Funktionenklassen herleiten. Da kann ich doch nicht das
Wachstumsverhalten jeder einzelnen Funktion betrachten. Deshalb mußte
ich die algebraisch Abhängigkeit der Funktionsterme mit der der
Funktionswerte in Zusammenhang bringen.
Da bezweifle ich aber, dass man aus diesem Zusammenhang etwas machen kann.
z.B. gibt es algebraisch unabhängige Funktionen, deren
Funktionswerte an derselben Stelle aber algebraisch abhängig sind.
Schau dir nur exp(z) und ln(z+1) an der Stelle z=0 an.
Also gilt nicht:

(f1, f2 algebraisch unabhängig => f1(z_0) und f2(z_0) algebraisch
unabhängig)

oder dazu äquivalent, es gilt nicht:

(f1(z_0) und f2(z_0) algebraisch abhängig => f1, f2 algebraisch abhängig)

Aloha
IV
2018-04-11 15:19:34 UTC
Permalink
Post by Torn Rumero DeBrak
Post by IV
Ich sehe das folgendermaßen. Ich will doch allgemeine Aussagen für
Funktionenklassen herleiten. Da kann ich doch nicht das
Wachstumsverhalten jeder einzelnen Funktion betrachten. Deshalb mußte ich
die algebraisch Abhängigkeit der Funktionsterme mit der der
Funktionswerte in Zusammenhang bringen.
Da bezweifle ich aber, dass man aus diesem Zusammenhang etwas machen kann.
z. B. gibt es algebraisch unabhängige Funktionen, deren Funktionswerte an
derselben Stelle aber algebraisch abhängig sind.
Schau dir nur exp(z) und ln(z+1) an der Stelle z=0 an.
(f1, f2 algebraisch unabhängig => f1(z_0) und f2(z_0) algebraisch
unabhängig)
(f1(z_0) und f2(z_0) algebraisch abhängig => f1, f2 algebraisch abhängig)
Das haben die forschenden Mathematiker in MathOverflow
https://mathoverflow.net/questions/296676/algebraic-independence-of-the-composition-of-functions
doch schon beantwortet: Die Funktionen dürfen in der Schnittmenge ihrer
Definitionsbereiche nicht konstant sein.
Deshalb gilt das allgemein für nichtkonstante holomorphe Funktionen über
einem Gebiet.
Und ich konnte doch schon "meinen" Satz aus dem ersten Thread herleiten.
Vielleicht läßt sich damit ja etwas anfangen.
Torn Rumero DeBrak
2018-04-11 23:00:35 UTC
Permalink
Post by IV
Post by Torn Rumero DeBrak
Post by IV
Ich sehe das folgendermaßen. Ich will doch allgemeine Aussagen für
Funktionenklassen herleiten. Da kann ich doch nicht das
Wachstumsverhalten jeder einzelnen Funktion betrachten. Deshalb mußte
ich die algebraisch Abhängigkeit der Funktionsterme mit der der
Funktionswerte in Zusammenhang bringen.
Da bezweifle ich aber, dass man aus diesem Zusammenhang etwas machen kann.
z. B. gibt es algebraisch unabhängige Funktionen, deren Funktionswerte
an derselben Stelle aber algebraisch abhängig sind.
Schau dir nur exp(z) und ln(z+1) an der Stelle z=0 an.
(f1, f2 algebraisch unabhängig => f1(z_0) und f2(z_0) algebraisch
unabhängig)
(f1(z_0) und f2(z_0) algebraisch abhängig => f1, f2 algebraisch abhängig)
Das haben die forschenden Mathematiker in MathOverflow
https://mathoverflow.net/questions/296676/algebraic-independence-of-the-composition-of-functions
doch schon beantwortet: Die Funktionen dürfen in der Schnittmenge ihrer
Definitionsbereiche nicht konstant sein.
Was hat das mit meiner Aussage zu tun? Ich habe keine konstante
Funktionen erwähnt. Was interpretierst du immer in das geschriebenen
Wort hinein?

Deshalb noch einmal meine Frage:
Weist du, wann zwei Funktionen gleich sind?

Und kannst du das hier bitte dann einmal aufschreiben.
Ich glaube nämlich, du weist es nicht, so wie du immer
am Thema vorbei antwortest.
Post by IV
Deshalb gilt das allgemein für nichtkonstante holomorphe Funktionen über
einem Gebiet.
Und ich konnte doch schon "meinen" Satz aus dem ersten Thread herleiten.
Vielleicht läßt sich damit ja etwas anfangen.
Ich bezog mich auf deine Aussage "Deshalb mußte ich die algebraisch
Abhängigkeit der Funktionsterme mit der der Funktionswerte in
Zusammenhang bringen."

Machst du das nun oder machst du das nicht, wenn MathOverflow dir davon
abrät?

Aloha
IV
2018-04-12 17:42:07 UTC
Permalink
Post by IV
Post by Torn Rumero DeBrak
Post by IV
Post by IV
Post by IV
Algebraische Unabhängigkeit
Und ist meine Definition auch richtig? Die ist für mich (jedenfalls
im Moment noch) wesentlich anschaulicher.
Die andere Richtung könnte haariger zu zeigen sein ... vielleicht mit
dem Satz über implizite Funktionen?
Womöglich müssen dann gewisse Punkte aus dem Defbereich genommen
werden.
Ich sehe das folgendermaßen. Ich will doch allgemeine Aussagen für
Funktionenklassen herleiten. Da kann ich doch nicht das
Wachstumsverhalten jeder einzelnen Funktion betrachten. Deshalb mußte
ich die algebraische Abhängigkeit der Funktionsterme mit der der
Funktionswerte in Zusammenhang bringen.
Da bezweifle ich aber, dass man aus diesem Zusammenhang etwas machen kann.
z. B. gibt es algebraisch unabhängige Funktionen, deren Funktionswerte
an derselben Stelle aber algebraisch abhängig sind.
Schau dir nur exp(z) und ln(z+1) an der Stelle z=0 an.
(f1, f2 algebraisch unabhängig => f1(z_0) und f2(z_0) algebraisch
unabhängig)
(f1(z_0) und f2(z_0) algebraisch abhängig => f1, f2 algebraisch abhängig)
Das haben die forschenden Mathematiker in MathOverflow
https://mathoverflow.net/questions/296676/algebraic-independence-of-the-composition-of-functions
doch schon beantwortet: Die Funktionen dürfen in der Schnittmenge ihrer
Definitionsbereiche nicht konstant sein.
Was hat das mit meiner Aussage zu tun? Ich habe keine konstante Funktionen
erwähnt. Was interpretierst du immer in das geschriebene Wort hinein?
Du sprichst von der Stelle z=0, und z_0 interpretiere ich auch als Stelle,
nicht als Variable.
In \mathbb{C} dürften die algebraischen Zahlen isoliert liegen, und damit
für holomorphe transzendente Funktionen über einem Gebiet auch die
"exceptional points".
Und mehr noch: f1 und f2 algebraisch unabhängig heißt, das gilt für f1(Z)
und f2(Z) für alle z aus der Schnittmenge Z der beiden Definitionsbereiche.
In jeder Umgebung zu einem algebraisch abhängigen Funktionswert liegt immer
auch ein algebraisch unabhängiger Funktionswert, weshalb immer auch f1(Z)
und f2(Z) algebraisch unabhängig sind - außer bei "irgendwo" in Z konstanten
Funktionen. Darauf weist der Kommentar in MathOverflow hin, und darauf, daß
nichtkonstante holomorphe Funktionen über einem Gebiet "nirgendwo" konstant
sind (nicht stückweise konstant, nicht lokal konstant und nicht konstant -
IV). Aber wegen meines Beweises des Satzes über die algebraische
Unabhängigkeit ganz oben vom Anfang dieser Diskussion hier will ich
demnächst mal hier nachfragen.
Deshalb noch einmal meine Frage: Weißt du, wann zwei Funktionen gleich
sind?
Ja. Und wir hatten das hier schon zwei, drei Mal aufgeschrieben.
Ich glaube nämlich, du weißt es nicht, so wie du immer am Thema vorbei
antwortest.
Willst Du mich denn auf einen konkreten Fehler bezüglich der Anwendung
dieser Definition hinweisen?
Ich bezog mich auf deine Aussage "Deshalb mußte ich die algebraische
Abhängigkeit der Funktionsterme mit der der Funktionswerte in Zusammenhang
bringen."
Machst du das nun oder machst du das nicht, wenn MathOverflow dir davon
abrät?
Ich weiß nicht, warum Du an dieser Stelle MathOverflow ins Spiel bringst. In
meinem Link wurde doch vom Beweis, den man "can easily see" gar nichts
preisgegeben.
Wie oben in diesem Post muß ich wahrscheinlich anstelle von f1(z) und f2(z)
setzen f1(Z) und f2(Z):
f1 und f2 algebraisch unabhängig <=> f1(Z) und f2(Z) algebraisch unabhängig.
Oder meinst Du etwas Anderes?
(Wenn Du das klar benennst geht alles viel schneller, brauche ich nicht
rumrätseln, Euch nicht langweilen und Euch nicht nerven.)
IV
2018-04-13 16:26:20 UTC
Permalink
"IV" schrieb im Newsbeitrag news:pao5pd$c9g$***@news.albasani.net...
"Torn Rumero DeBrak" schrieb im Newsbeitrag ...
Nun habe ich mich auch diesmal wieder von Dir ins Bockshorn jagen lassen und
war unsicher geworden. "f1 und f2 algebraisch unabhängig => f1(Z) und f2(Z)
algebraisch unabhängig" ist ja noch falscher. Es soll schon so heißen wie
oben: "f1 und f2 algebraisch unabhängig => f1(z) und f2(z) algebraisch
unabhängig". f1(z) und f2(z) sind die Funktionsterme, z also die Variable.
Torn Rumero DeBrak
2018-04-13 17:41:54 UTC
Permalink
"Torn Rumero DeBrak"  schrieb im Newsbeitrag ...
Nun habe ich mich auch diesmal wieder von Dir ins Bockshorn jagen lassen
War nicht meine Absicht.
und war unsicher geworden. "f1 und f2 algebraisch unabhängig => f1(Z)
und f2(Z) algebraisch unabhängig" ist ja noch falscher. Es soll schon so
heißen wie oben: "f1 und f2 algebraisch unabhängig => f1(z) und f2(z)
algebraisch unabhängig". f1(z) und f2(z) sind die Funktionsterme, z also
die Variable.
Und was soll jetzt "f1(z) und f2(z) algebraisch unabhängig" bedeuten im
Gegensatz zu "f1 und f2 algebraisch unabhängig" ? Ich meinte (und
schrieb) "f1 und f2 algebraisch unabhängig => f1(z_0) und f2(z_0)
algebraisch unabhängig" für eine Stelle z_0, nicht für eine Variable z.

Du schmeist an den unmöglichsten Stellen die Variablen und konstante
Funktionsargumente sowie Funktionsnamen und Funktionswerte
durcheinander.


Gehen bei dir die Allquantoren womöglich verloren?

Aloha
IV
2018-04-13 20:59:53 UTC
Permalink
Es soll schon so heißen wie oben: "f1 und f2 algebraisch unabhängig =>
f1(z) und f2(z) algebraisch unabhängig". f1(z) und f2(z) sind die
Funktionsterme, z also die Variable.
Und was soll jetzt "f1(z) und f2(z) algebraisch unabhängig" bedeuten im
Gegensatz zu "f1 und f2 algebraisch unabhängig" ? Ich meinte (und schrieb)
"f1 und f2 algebraisch unabhängig => f1(z_0) und f2(z_0) algebraisch
unabhängig" für eine Stelle z_0, nicht für eine Variable z.
...
Du schmeißt an den unmöglichsten Stellen die Variablen und konstante
Funktionsargumente sowie Funktionsnamen und Funktionswerte durcheinander.
Gehen bei dir die Allquantoren womöglich verloren?
f1(z) und f2(z) sind die Funktionsterme - Terme wie sie in einer Gleichung
vorkommen, mit z als Gleichungsvariable.
Beispiel exp und ln. Die Funktionen exp und ln über nicht diskretem
Definitionsbereich sind algebraisch unabhängig voneinander über dem
Koeffizientenkörper \mathbb{C}, und auch deren Funktionsterme exp(z) und
ln(z) - man kann sie nicht durch algebraische Operationen (Anwenden einer
algebraischen Funktion mit Koeffizienten aus dem Körper \mathbb{C})
ineinander umwandeln. Einzelne Funktionswerte von exp und ln können aber
durchaus algebraisch abhängig voneinander über dem Koeffizientenkörper
\mathbb{C} sein, ja die Funktionswerte zweier Funktionen über der nicht
leeren Schnittmenge ihrer Definitionsbereiche müssen sogar paarweise
algebraisch abhängig voneinander über dem Koeffizientenkörper \mathbb{C}
sein, denn es sind komplexe Zahlen.

Für meine Beweise möchte ich nicht die
umfangreichen/unvollständigen/heuristischen/unübersichtlichen Algorithmen
der Termvereinfachung (entweder in der Sprache der Funktionsbezeichner oder
in der Sprache der Funktionsterme) anwenden. Deshalb meine eigene Definition
der algebraischen Unabh�ngigkeit hier: news:pae05o$lhu$***@news.albasani.net
Und Detlef hatte ja die Korrektheit meiner Definition bestätigt.
Es kann sein, daß es eine körpertheoretische Definition der algebraischen
Unabhängigkeit von Funktionen gibt, ich habe aber noch nichts Passendes
gefunden.
Torn Rumero DeBrak
2018-04-16 18:13:27 UTC
Permalink
Post by IV
Es soll schon so heißen wie oben: "f1 und f2 algebraisch unabhängig =>
f1(z) und f2(z) algebraisch unabhängig". f1(z) und f2(z) sind die
Funktionsterme, z also die Variable.
Und was soll jetzt "f1(z) und f2(z) algebraisch unabhängig" bedeuten
im Gegensatz zu "f1 und f2 algebraisch unabhängig" ? Ich meinte (und
schrieb) "f1 und f2 algebraisch unabhängig => f1(z_0) und f2(z_0)
algebraisch unabhängig" für eine Stelle z_0, nicht für eine Variable z.
...
Du schmeißt an den unmöglichsten Stellen die Variablen und konstante
Funktionsargumente sowie Funktionsnamen und Funktionswerte durcheinander.
Gehen bei dir die Allquantoren womöglich verloren?
f1(z) und f2(z) sind die Funktionsterme - Terme wie sie in einer
Gleichung vorkommen, mit z als Gleichungsvariable.
Beispiel exp und ln.
Also ist exp(z) nur ein anderer Name für den Term

sum_i=0^oo (z^i / i! ),

und ln(z) nur eine andere, verkürzte Bezeichnung für den Term

sum_i=1^oo ( (-1)^(i-1) * (z-1)^i / i )

richtig?
oo
----
\
mit der Schreibweise sum_i=1^oo für /
----
i=1


Aloha
IV
2018-04-17 17:42:55 UTC
Permalink
Post by Torn Rumero DeBrak
Post by IV
Es soll schon so heißen wie oben: "f1 und f2 algebraisch unabhängig =>
f1(z) und f2(z) algebraisch unabhängig". f1(z) und f2(z) sind die
Funktionsterme, z also die Variable.
f1(z) und f2(z) sind die Funktionsterme - Terme wie sie in einer
Gleichung vorkommen, mit z als Gleichungsvariable.
...
Beispiel exp und ln.
Also ist exp(z) nur ein anderer Name für den Term
sum_i=0^oo (z^i / i! ),
und ln(z) nur eine andere, verkürzte Bezeichnung für den Term
sum_i=1^oo ( (-1)^(i-1) * (z-1)^i / i )
richtig?
Ich denke mal, ja. Es ist ein Term mit der Variablen z. Wenn man für die
Variable z eine Stelle z0 setzt, ergibt sich ein Funktionswert, wenn man für
die Variable eine Menge von Stellen setzt, ergibt sich eine Menge von
Funktionswerten.
Es geht um Funktionsterme, z. B. um den Term z * exp(z), der Funktionsterm
einer Funktion id * exp ist, und um Gleichungsterme, z. B. in der Gleichung
z * exp(z) - 1 = 0. Dies ist die Nullstellengleichung einer Funktion id *
exp +1. z ist eine Variable, in Bestimmungsgleichungen auch Lösungsvariable
oder Unbekannte genannt.
IV
2018-04-17 18:37:15 UTC
Permalink
Post by IV
z * exp(z) - 1 = 0
Korrektur: Nullstellengleichung einer Funktion id * exp - 1

Wenn man für die Termvariable nichts einsetzt, ist es einfach nur ein Term.
Mit Termen kann man z. B. Termumformung und Termvereinfachung betreiben.
Einem Term alleine kann man seine Verwendung nicht ansehen - man muß sie mit
angeben, z. B. durch das Festlegen von z.
Jens Kallup
2018-04-17 19:00:56 UTC
Permalink
Post by IV
um den Term z * exp(z)
Also ich verstehe Eure Hochspannenden Diskussionen nicht.
Ein Term, eine Funktion, eine Variable, ein Funktionswert....
hin und her. -___- <- Quark.

A) Wenn in einer Gleichung a + b + c verwendet werden,
so haben a, b, und c eine andere Bedeutung,
Zum Beispiel kann das Symbol a auch ein Term sein, der zusammen
gefaßt wurde, um nicht stetig bekannte Größen/Rechenschritte
zu machen - gerade in der Computer-Ecke, bei der es um Schnellig-
keit bei bestimmten Berechnungen geht...

B) können a, b, oder c _eine_ Zahl zu einer Konstanten gehören.
Als Allgemein-Regel werden Konstanten auf Null (0) gesetzt.
Konstanten können aber auch zu einen Funktionwert gehören:
wie zum Beispiel die Kreiszahl (die eigentlich dynamisch ist,
wurde auf einen konstanten Wert, der für allgemeine Berechnungen
genutzt wird) Pi = 3,14 .
Oder die Euler-Zahl, gilt auch als Konstante, jedoch mit einen
Wert von 2,718 .
Oder geht man hinab ins kleinere Mathematik Geschehen:
_Ein_ Meter ist konstantiert durch 100 Zentimeter.
All diese normalisierten und standardisierten "Neufassungen" der
Mathematik würden noch vor geraumer Zeit anders aufgefast.

C) Teils entstehen Terme durch verschiedene mathematische Berechnungen
durch Lemmatas - sogenannte Teilschritte.

D) Man sollte stets Wissen, welcher Weg zu Begehen ist, um wieder auf
den Anfangspunkt zu kommen.
In der Mathematik ist dies als Umkehrfunktion oder besser bekannt
durch "Probe" gekennzeichnet.
Zum Beispiel ist die Umkehrfuntion von der:
die Addition = die Subtraktion, oder
die Multiplikation = die Division, etc. ...

Um zum Beispiel eine stellte Gleichung der Form:
z * exp(z) = 0

zu lösen, ist es Anfangs egal was z ist.
Ob das nun eine statische/konstante Variable, eine durchlaufende
Variable ist oder einfach nur ein Symbol, das zu einen Objekt gehört
(Objekte sind, wie ich mittlerweile gelernt habe nummerische Zeichen).

Es ist auch Egal was am Ende heraus kommen soll - im obigen Beispiel
die Nullstelle = 0 - kann auch gefragt werden, wann die Gleichung 1
erreicht - zum Beispiel in einen Durchlauf/Schleife/Loop ...

Das kann bei Prüfungen (nicht nur speziell Mathe) abgefragt werden:
Formen Sie Bitte folgende Gleichung so um, das .... etc.

So würde die Umkehrfunktion von: f(x) = z * exp(z)
sich nicht so einfach auflösen, da hier das Lambert Verfahren angewandt
werden müsste.
Füd die Lambert-Wert-Zeichen-Funktion wird der einfachhalt halber W
gesetzt:

Für die Funktion W(x) gilt:

x >= -1 / e

Herleitung:

W(- 1/2 * Pi) = 1/2 * i * Pi
W(-e^(-1) = -1
W(0) = 0
W(1) = 0,567 <= die ist auch die Omega-Konstante oder: goldener Schnitt

Nun überlegt man, was die Umkehrfunktion von exp ist...
Okay, klar: ln

Dann würde man für: ln[ z/(W(z) ] = W(z)
erhalten.


So könnte man dies ausschauen:

1. Lemma/Term: exp(z) = ln[ z/(W(z) ] = W(z)
2. Lemma/Term: z * W(z) = 0

Als Lösung erhält man dann für 0, was nach Definition angenommen werden
kann (x >= -1 / e) :

z = 0

0 * W(z) = 0
0 * W(0) = 0
0 * 0 = 0 [ok]

Fazit: für z * exp(z) = 0
gibt es _keine_ Umkehrfunktion! oder kann man in der Mathematik
die Null einfach zu rumrollern lassen ;-) ?

Gruß
Jens
Detlef Müller
2018-04-17 20:11:59 UTC
Permalink
[...]
Post by IV
Post by Torn Rumero DeBrak
Post by IV
Beispiel exp und ln.
Also ist exp(z) nur ein anderer Name für den Term
sum_i=0^oo (z^i / i! ),
und ln(z) nur eine andere, verkürzte Bezeichnung für den Term
sum_i=1^oo ( (-1)^(i-1) * (z-1)^i / i )
richtig?
Ich denke mal, ja.
[...]

Sind dann die Terme

sum_i=0^oo (z^i / i! )

und

lim_{n->oo}(1+z/n)^n

algebraisch unabhängig?

Gruß,
Detlef
Jens Kallup
2018-04-17 22:26:40 UTC
Permalink
Post by Detlef Müller
[...]
Post by IV
Post by Torn Rumero DeBrak
Post by IV
Beispiel exp und ln.
Also ist exp(z) nur ein anderer Name für den Term
sum_i=0^oo (z^i / i! ),
und ln(z) nur eine andere, verkürzte Bezeichnung für den Term
sum_i=1^oo ( (-1)^(i-1) * (z-1)^i / i )
richtig?
Ich denke mal, ja.
[...]
Sind dann die Terme
  sum_i=0^oo (z^i / i! )
1. Wenn oo für unendlich steht, dann kann dort jedes ZeichenObjekt
stehen.
Da 0 x 0 = 0 ergibt (vielleicht der Ursprung des Zeichens oo ?);
So ergibt 0 Hoch oo wieder Null: 0 x 0 x 0 x 0, ...

2. Zahlen nicht erklärt. Um derartige Größen zuzulassen, wurden
sogenannte imaginäre Zahlen eingeführt.
Die Quadratwurzel mit einem negativen Radikanden ist ein imaginäre
Zahl.
Da keine reelle Zahl existiert, deren Quadrat -1 ist, erweitert man
den Zahlenbegriff um die imaginäre Einheit i = Wurzel(-1).
Diese Einheit führte L. Euler ein.

Es gilt also: i^2 = -1

3. z^i = z^(-1)

4. i! = 1

A) !!! Jetzt aufpassen:

Potenzen können nicht mit einen negativen Wert (-1) potenziert
werden.
Man muss daher die inverse Bilden.
Dies geschiet, indem man den Kehrwert des Ausdrucks z^i bildet.
Im grundegenommen entspricht dies dem Kehrwert einer Division:

Man formt also die innere Gleichung: z^(i) / !i mittels
Reziproke um:

L1 = !i * z^(i) | 1. Lemma
!!!

A) Jetzt setzen wir unser Gedankenspiel weiter fort, und denken uns für
i die 1 (Eins) und z = 0 ...

Dann ergibt sich aus L1:

L2: i! = 1 und
L3: z^(i) = z^(1) = 0^1 = 1

Wir führen beide Lemmatas zusammen:

L2 * L3 = L4
L4 = 1

B) wie oben beschrieben, ergibt 0 Hoch unendlich gleich 0.
dann greife ich dies mal wieder auf, und setze das Lemma 5

L5 = 0

Dann ergibt sich eine Summengleichung:

sum_0 = (L5 * L4)
sum_0 = 0

sum_1 = (L5 * L4)
sum_1 = 0

...
sum_oo
Post by Detlef Müller
und
 lim_{n->oo}(1+z/n)^n
1. L1: (1 + z/n)^n | z = 0
(1 + 0/n)^n | n = 1 Achtung: 0 division 0 geht hier nicht !!!
(1 + 0/1)^1
(1 + 0 )^1
(1 )^1 = 1

2. L2: lim_{1 gegen unendlich}

3. Probe: L1 = 1
L2 = 1

:= L1 * L2 = 1 oder:


L1 = 1
L2 = 2

:= L1 * L2 = 2

Beweis: siehe obige Probe: 1,2, ... , oo
Post by Detlef Müller
algebraisch unabhängig?
Antwort:

50 zu 50 - warum:
Man kann leicht den Anschein erwecken, das die 2 Gleichenungen
untereinander Gleich sind.
Sind sie aber *nicht* !!!
Begründung:
Erst mit Kenntnise der Mathematik lassen sich Entscheidungen treffen.
Daher sollte man Entscheidungen - wie im Business - gut überdenken,
bevor man Entschlüße Diskutiert oder trifft.
Allerdings:
Ist der Rechenaufwand vielleicht gleich bzw leichter zu erwägen, wenn
man sein Wissen einsetzt bzw. seriöse Quellen damit einsetzt eben
diese Entscheidung(en) zu treffen.

Fazit:
1. algebraisch gleich = 50%
2. unabhängig von einander = 50%

Nummer IV will anscheiend andere Helferlein damit beauftragen, seine
Ausgedachten Formeln zu bearbeiten???

Ich mein, ist ja nix daran auszusetzen, aber dann immer hinzu zu
Schreiben -> Ja, verstanden, mit Quellenangabe
macht irgendwie in meinen Augen den Eindruck, das Nummber IV überhaupt
keinen Plan hat von der Materie.

So habe ich noch keine Mitglieder unserer Einheit erlebt.

Naja, ich will nicht sticheln, regt mich nur ein wenig auf, wenn man mit
halben Herzens an die Sache geht und Halbwissen verbreitet.

Ihr könnt mich jetzt auch ausmeckern; heißt ja: die Konkurrenz schläft
nie!

Nein, Spaß
Post by Detlef Müller
Gruß,
  Detlef
Gruß Jens
Torn Rumero DeBrak
2018-04-18 08:10:36 UTC
Permalink
Post by Jens Kallup
Post by Detlef Müller
[...]
Post by IV
Post by Torn Rumero DeBrak
Post by IV
Beispiel exp und ln.
Also ist exp(z) nur ein anderer Name für den Term
sum_i=0^oo (z^i / i! ),
und ln(z) nur eine andere, verkürzte Bezeichnung für den Term
sum_i=1^oo ( (-1)^(i-1) * (z-1)^i / i )
richtig?
Ich denke mal, ja.
[...]
Sind dann die Terme
   sum_i=0^oo (z^i / i! )
1. Wenn oo für unendlich steht, dann kann dort jedes ZeichenObjekt
   stehen.
Du müsstest schon mein Posting bis zu Ende lesen. Ich hatte noch de
Hinweis geschrieben:

oo
----
\
mit der Schreibweise sum_i=1^oo für /
----
i=1


Somit ist i als Summenindex und nicht als imaginäre Einheit zu verstehen
und in sum_i=0^oo steht keine Potenz sondern eine untere und
oberere Summengrenze. Diese Schreibweise ist in der ASCII-Welt aber
bekannt.

Dein folgender Vortrag ist wegen dieses Missverständnisses off-Topic.
Post by Jens Kallup
   Da 0 x 0 = 0 ergibt (vielleicht der Ursprung des Zeichens oo ?);
   So ergibt 0 Hoch oo wieder Null: 0 x 0 x 0 x 0, ...
2. Zahlen nicht erklärt. Um derartige Größen zuzulassen, wurden
   sogenannte imaginäre Zahlen eingeführt.
   Die Quadratwurzel mit einem negativen Radikanden ist ein imaginäre
   Zahl.
   Da keine reelle Zahl existiert, deren Quadrat -1 ist, erweitert man
   den Zahlenbegriff um die imaginäre Einheit i = Wurzel(-1).
   Diese Einheit führte L. Euler ein.
   Es gilt also: i^2 = -1
3. z^i  = z^(-1)
4. i!   = 1
       Potenzen können nicht mit einen negativen Wert (-1) potenziert
       werden.
       Man muss daher die inverse Bilden.
       Dies geschiet, indem man den Kehrwert des Ausdrucks z^i bildet.
       Man formt also die innere Gleichung: z^(i) / !i  mittels
       L1 = !i * z^(i)  | 1. Lemma
   !!!
A) Jetzt setzen wir unser Gedankenspiel weiter fort, und denken uns für
   i die 1 (Eins) und z = 0 ...
   L2: i! = 1                   und
   L3: z^(i) = z^(1) = 0^1 = 1
   L2 * L3 = L4
   L4 = 1
B) wie oben beschrieben, ergibt 0 Hoch unendlich gleich 0.
   dann greife ich dies mal wieder auf, und setze das Lemma 5
   L5 = 0
   sum_0 = (L5 * L4)
   sum_0 = 0
   sum_1 = (L5 * L4)
   sum_1 = 0
   ...
   sum_oo
Post by Detlef Müller
und
  lim_{n->oo}(1+z/n)^n
1. L1: (1 + z/n)^n   | z = 0
       (1 + 0/n)^n   | n = 1  Achtung: 0 division 0 geht hier nicht !!!
       (1 + 0/1)^1
       (1 + 0  )^1
       (1      )^1 = 1
2. L2: lim_{1 gegen unendlich}
3. Probe:  L1 = 1
           L2 = 1
           L1 = 1
           L2 = 2
   := L1 * L2 = 2
Beweis: siehe obige Probe: 1,2, ... , oo
Post by Detlef Müller
algebraisch unabhängig?
Man kann leicht den Anschein erwecken, das die 2 Gleichenungen
untereinander Gleich sind.
Sind sie aber *nicht* !!!
Erst mit Kenntnise der Mathematik lassen sich Entscheidungen treffen.
Daher sollte man Entscheidungen - wie im Business - gut überdenken,
bevor man Entschlüße Diskutiert oder trifft.
Ist der Rechenaufwand vielleicht gleich bzw leichter zu erwägen, wenn
man sein Wissen einsetzt bzw. seriöse Quellen damit einsetzt eben
diese Entscheidung(en) zu treffen.
1. algebraisch gleich      = 50%
2. unabhängig von einander = 50%
Nummer IV will anscheiend andere Helferlein damit beauftragen, seine
Ausgedachten Formeln zu bearbeiten???
Ich mein, ist ja nix daran auszusetzen, aber dann immer hinzu zu
Schreiben -> Ja, verstanden, mit Quellenangabe
macht irgendwie in meinen Augen den Eindruck, das Nummber IV überhaupt
keinen Plan hat von der Materie.
So habe ich noch keine Mitglieder unserer Einheit erlebt.
Naja, ich will nicht sticheln, regt mich nur ein wenig auf, wenn man mit
halben Herzens an die Sache geht und Halbwissen verbreitet.
Ihr könnt mich jetzt auch ausmeckern; heißt ja: die Konkurrenz schläft
nie!
Nein, Spaß
Post by Detlef Müller
Gruß,
   Detlef
Gruß Jens
Aloha
Jens Kallup
2018-04-18 08:34:03 UTC
Permalink
Post by Torn Rumero DeBrak
Somit ist i als Summenindex und nicht als imaginäre Einheit zu verstehen
und in sum_i=0^oo steht keine Potenz sondern eine untere und
oberere Summengrenze. Diese Schreibweise ist in der ASCII-Welt aber
bekannt.
Bitte entschuldige mich, ich bin Windows gecrankter - samt grafischer
Symbole und Co.
Da hier kein Grafikposting gern sehen wird, habe ich nun zur Kenntniss
genommen. Ich muss noch viel lernen in der ASCII Welt, obwohl ich aus
der Zeit des BTX komme... naja, egal.

Deinen abgeschnippelten Satz entnehme ich, das hier keine Hausaufgaben
gemacht werden sollen.
Aber sei mal ehrlich, IV ist doch schon schwerer Tobak - oder?

Gruß, Jens
IV
2018-04-18 16:09:48 UTC
Permalink
Post by Detlef Müller
Post by Torn Rumero DeBrak
Post by IV
Beispiel exp und ln.
Also ist exp(z) nur ein anderer Name für den Term
sum_i=0^oo (z^i / i! ),
und ln(z) nur eine andere, verkürzte Bezeichnung für den Term
sum_i=1^oo ( (-1)^(i-1) * (z-1)^i / i )
richtig?
Ich denke mal, ja. ...
Sind dann die Terme
sum_i=0^oo (z^i / i! )
und
lim_{n->oo}(1+z/n)^n
algebraisch unabhängig?
Ich denke mal, über demselben zugrundegelegten Definitionsbereich der
Funktionen f1, f2 und der Terme ergibt sich aus der algebraischen
Unabhängigkeit (über einem Körper K) die algebraische Unabhängigkeit (über
dem Körper K) der beiden Terme.
(Ihr braucht mir keine mathematischen Fragen stellen - ich bin hier nicht
der Mathematiker, sondern derjenige der Fragen hat und einfach nur Antworten
benötigt.)
H0Iger SchuIz
2018-04-18 17:42:45 UTC
Permalink
Post by IV
(Ihr braucht mir keine mathematischen Fragen stellen - ich bin hier nicht
der Mathematiker, sondern derjenige der Fragen hat und einfach nur Antworten
benötigt.)
Selbst denken (mit gewissen Anregungen) ist keine Option?

hs
IV
2018-04-19 18:34:35 UTC
Permalink
Post by H0Iger SchuIz
Post by IV
(Ihr braucht mir keine mathematischen Fragen stellen - ich bin hier nicht
der Mathematiker, sondern derjenige der Fragen hat und einfach nur
Antworten benötigt.)
Selbst denken (mit gewissen Anregungen) ist keine Option?
Doch, aber Ihr stellt mir ja nur Fragen, die Ihr viel besser beantworten
könntet. Das hält nur auf.
H0Iger SchuIz
2018-04-20 06:23:14 UTC
Permalink
Post by IV
Post by H0Iger SchuIz
Post by IV
(Ihr braucht mir keine mathematischen Fragen stellen - ich bin hier nicht
der Mathematiker, sondern derjenige der Fragen hat und einfach nur
Antworten benötigt.)
Selbst denken (mit gewissen Anregungen) ist keine Option?
Doch, aber Ihr stellt mir ja nur Fragen, die Ihr viel besser beantworten
könntet. Das hält nur auf.
Einfach ignorieren, was einen nicht interessiert.

hs
Christian Gollwitzer
2018-04-19 06:06:33 UTC
Permalink
Post by IV
Post by Detlef Müller
Post by Torn Rumero DeBrak
Post by IV
Beispiel exp und ln.
Also ist exp(z) nur ein anderer Name für den Term
sum_i=0^oo (z^i / i! ),
und ln(z) nur eine andere, verkürzte Bezeichnung für den Term
sum_i=1^oo ( (-1)^(i-1) * (z-1)^i / i )
richtig?
Ich denke mal, ja. ...
Sind dann die Terme
  sum_i=0^oo (z^i / i! )
und
 lim_{n->oo}(1+z/n)^n
algebraisch unabhängig?
Ich denke mal, über demselben zugrundegelegten Definitionsbereich der
Funktionen f1, f2 und der Terme ergibt sich aus der algebraischen
Unabhängigkeit (über einem Körper K) die algebraische Unabhängigkeit
(über dem Körper K) der beiden Terme.
(Ihr braucht mir keine mathematischen Fragen stellen - ich bin hier
nicht der Mathematiker, sondern derjenige der Fragen hat und einfach nur
Antworten benötigt.)
Auf die Gefahr hin, dass Du mir das nicht danken wirst, bin ich mal
expliziter als Tom. Er möchte darauf hinweisen, dass Gleichheit für
Terme kein üblicher Begriff ist, bzw. er möchte wissen, was Du darunter
verstehst. Als einfaches Beispiel hat er zwei mögliche Terme
hingeschrieben, die ausgewertet beide exp(z) ergeben. SInd die jetzt
gleich, weil sie punktweise gleich sind, d.h. für jedes z das man
einsetzen kann kommt dasselbe heraus, oder sind sie nicht gleich, weil
die Buchstabenfolge bzw. die Berechnungsvorschrift eine andere ist?

Ein einfacheres Beispiel wäre:
Sind die Terme T1:="(x^2-y^2)/(x-y)" und T2:="(x+y)" gleich?
Daran erkennt man gleich noch ein weiteres Problem, der Term T1 ist für
x=y nicht definiert, der zweite schon. Punktweise kann man sie dort also
nicht vergleichen. Schränkt man die Gleichheit nur darauf ein, wo beide
Terme definiert sind, dann ergibt sich die perverse Sache, dass ein
Term, der nirgends definiert ist, mit allen anderen gleich ist. Und
somit hat man die Transzendenz der Gleichheit verloren.

Und weil Du Dich offenbar für Computeralgebra interessierst, ich denke
Du kennst das Nullvergleichsproblem - man kann keinen Algorithmus
finden, der sicher entscheidet, wann zwei Terme gleich sind. Ich
fürchte, Dein "Umkehrungsproblem" lässt sich darauf zurückführen, auch
wenn ich das jetzt nicht beweisen kann, und ist damit unlösbar.


Christian
Christian Gollwitzer
2018-04-19 06:15:43 UTC
Permalink
Post by Christian Gollwitzer
Und weil Du Dich offenbar für Computeralgebra interessierst, ich denke
Du kennst das Nullvergleichsproblem - man kann keinen Algorithmus
finden, der sicher entscheidet, wann zwei Terme gleich sind. Ich
fürchte, Dein "Umkehrungsproblem" lässt sich darauf zurückführen, auch
wenn ich das jetzt nicht beweisen kann, und ist damit unlösbar.
Ach ja, ich habe einen simplen Beweis gefunden: Deine Funktionenklasse
enthält mit Sicherheit alle Polynome, sonst wäre sie komplett nutzlos.
Für Polynome bis Grad 4 gibt es explizite Lösungsformeln, darüber
beweisbar nicht. Also kannst Du keine sinnvolle Funktionenklasse finden,
wo jede Funktion elementar lösbar ist.

Viel Erfolg,

Christian
IV
2018-04-19 20:12:24 UTC
Permalink
Post by Christian Gollwitzer
Post by Christian Gollwitzer
Und weil Du Dich offenbar für Computeralgebra interessierst, ich denke Du
kennst das Nullvergleichsproblem - man kann keinen Algorithmus finden,
der sicher entscheidet, wann zwei Terme gleich sind. Ich fürchte, Dein
"Umkehrungsproblem" lässt sich darauf zurückführen, auch wenn ich das
jetzt nicht beweisen kann, und ist damit unlösbar.
Ach ja, ich habe einen simplen Beweis gefunden: Deine Funktionenklasse
enthält mit Sicherheit alle Polynome, sonst wäre sie komplett nutzlos. Für
Polynome bis Grad 4 gibt es explizite Lösungsformeln, darüber beweisbar
nicht. Also kannst Du keine sinnvolle Funktionenklasse finden, wo jede
Funktion elementar lösbar ist.
Ritts Struktursatz gibt an, welche Struktur der Funktionsterm einer
(elementaren) Funktionen haben muß, damit die Funktion eine Umkehrfunktion
in der Klasse der Elementaren Funktionen hat, wenn sie bijektiv ist.
IV
2018-04-19 19:59:09 UTC
Permalink
Post by Christian Gollwitzer
Post by IV
Post by Detlef Müller
Post by Torn Rumero DeBrak
Also ist exp(z) nur ein anderer Name für den Term
sum_i=0^oo (z^i / i! ),
und ln(z) nur eine andere, verkürzte Bezeichnung für den Term
sum_i=1^oo ( (-1)^(i-1) * (z-1)^i / i )
richtig?
Ich denke mal, ja. ...
Sind dann die Terme
sum_i=0^oo (z^i / i! )
und
lim_{n->oo}(1+z/n)^n
algebraisch unabhängig?
Ich denke mal, über demselben zugrundegelegten Definitionsbereich der
Funktionen f1, f2 und der Terme ergibt sich aus der algebraischen
Unabhängigkeit (über einem Körper K) die algebraische Unabhängigkeit
(über dem Körper K) der beiden Terme.
Auf die Gefahr hin, dass Du mir das nicht danken wirst, bin ich mal
expliziter als Tom.
(Keine Ahnung wie Du darauf kommst. Mit Testfragen komme ich nicht klar, mit
Hinweisen und Antworten schon.)
Post by Christian Gollwitzer
Er möchte darauf hinweisen, dass Gleichheit für Terme kein üblicher
Begriff ist, bzw. er möchte wissen, was Du darunter verstehst. Als
einfaches Beispiel hat er zwei mögliche Terme hingeschrieben, die
ausgewertet beide exp(z) ergeben. SInd die jetzt gleich, weil sie
punktweise gleich sind, d.h. für jedes z das man einsetzen kann kommt
dasselbe heraus, oder sind sie nicht gleich, weil die Buchstabenfolge bzw.
die Berechnungsvorschrift eine andere ist?
Warum schreibt er das dann nicht?
Ich muß die algebraische Unabhängigkeit von Funktionen betrachten.
Funktionen sind durch ihren Definitionsbereich und ihre Zuordnungsvorschrift
mit dem Funktionsterm beschrieben. Zwischen Funktion und (DB +
Funktionsvariable + Funktionsterm) scheint mir eine Bijektion zu bestehen.
Post by Christian Gollwitzer
Sind die Terme T1:="(x^2-y^2)/(x-y)" und T2:="(x+y)" gleich?
Daran erkennt man gleich noch ein weiteres Problem, der Term T1 ist für
x=y nicht definiert, der zweite schon. Punktweise kann man sie dort also
nicht vergleichen. Schränkt man die Gleichheit nur darauf ein, wo beide
Terme definiert sind, dann ergibt sich die perverse Sache, dass ein Term,
der nirgends definiert ist, mit allen anderen gleich ist. Und somit hat
man die Transzendenz der Gleichheit verloren.
Beide Terme repräsentieren unterschiedliche Funktionen.
Post by Christian Gollwitzer
Und weil Du Dich offenbar für Computeralgebra interessierst, ich denke Du
kennst das Nullvergleichsproblem - man kann keinen Algorithmus finden, der
sicher entscheidet, wann zwei Terme gleich sind. Ich fürchte, Dein
"Umkehrungsproblem" lässt sich darauf zurückführen, auch wenn ich das
jetzt nicht beweisen kann, und ist damit unlösbar.
Ich möchte nur den, ich nenne ihn Struktursatz, von Ritt, der für Elementare
Funktionen gilt, auf allgemeinere Funktionenklassen verallgemeinern. Ich
nehme mal an, da spielt der Satz von Richardson keine Rolle.
Hans Crauel
2018-04-21 20:43:47 UTC
Permalink
IV schrieb
Post by IV
Funktionen sind durch ihren Definitionsbereich und ihre Zuordnungsvorschrift
mit dem Funktionsterm beschrieben. Zwischen Funktion und (DB +
Funktionsvariable + Funktionsterm) scheint mir eine Bijektion zu bestehen.
Nein. Im allgemeinen gibt es keinen "Funktionsterm".
Nimm etwa den Satz ueber implizite Funktionen, der die Existenz
einer (zudem differenzierbaren) Funktion postuliert. Einen
"Funktionsterm" gibt es da nicht.

Oder nimm ein Polynom fuenften Grades mit reellen (oder auch
komplexen) Koeffizienten,

(*) P(z) = az^5 + bz^4 + xz^3 + dx^2 +ex + d,

betrachte die dadurch induzierte komplexe Funktion und definiere,
fuer eine feste Wahl von a,b,d und e, die Funktion

x |-> max {Re(lambda) : lambda Nullstelle von (*)},

also das Maximum einer von x abhaengigen, nichtleeren Menge
reeller Zahlen.
Das gibt eine auf ganz R definierte reellwertige Funktion, ganz
ohne dass ein "Funktionsterm" auftaucht.
Fragen waeren nun, welche Eigenschaften diese Funktion hat, also
ob sie stetig ist (wohl ja), ob sie differenzierbar ist (i.a.
eher nein). Antworten auf solche Fragen moechte man erhalten,
ohne auch die Spur einer Chance zu haben, dort einen
"Funktionsterm" zu bestimmen.

Derart definierte Funktionen tauchen in mathematischen (wie auch
in Anwendungs-) Zusammenhaengen um ein Vielfaches haeufiger auf
als Funktionen, die durch einen "Funktionsterm" gegeben sind,
die sind eher langweilig.

Hans
IV
2018-04-22 18:58:25 UTC
Permalink
Post by Hans Crauel
Post by IV
Funktionen sind durch ihren Definitionsbereich und ihre
Zuordnungsvorschrift mit dem Funktionsterm beschrieben. Zwischen Funktion
und (DB + Funktionsvariable + Funktionsterm) scheint mir eine Bijektion
zu bestehen.
Nein. Im allgemeinen gibt es keinen "Funktionsterm".
Da ich in meinem Projekt die Funktionen über ihren Funktionsterm handhaben
muß, muß ich korrigieren:
M e i n e Funktionen sind durch ihren Definitionsbereich und ihre
Zuordnungsvorschrift mit dem Funktionsterm beschrieben. Zwischen Funktion
und (DB + Funktionsvariable + Funktionsterme) scheint mir eine Bijektion zu
bestehen.
H0Iger SchuIz
2018-04-23 07:23:36 UTC
Permalink
Post by IV
Post by Hans Crauel
Post by IV
Funktionen sind durch ihren Definitionsbereich und ihre
Zuordnungsvorschrift mit dem Funktionsterm beschrieben. Zwischen Funktion
und (DB + Funktionsvariable + Funktionsterm) scheint mir eine Bijektion
zu bestehen.
Nein. Im allgemeinen gibt es keinen "Funktionsterm".
Da ich in meinem Projekt die Funktionen über ihren Funktionsterm handhaben
muß,
Beziehen sich dann also die Sätze, die dieses "Projekt" hervorbringen
wird auf "Funktionen, die mittels Funktionsterm beschrieben werden"?
Oder wie muss man sich das vorstellen?
Post by IV
M e i n e Funktionen sind durch ihren Definitionsbereich und ihre
Zuordnungsvorschrift mit dem Funktionsterm beschrieben.
Was immer das nun sein soll. Für eine neue Art von Objekten sollte man
dann aber auch einen neuen Namen verwenden. Den Begriff "Funktion"
umzudefinieren, bringt's nicht.
Post by IV
Zwischen Funktion
und (DB + Funktionsvariable + Funktionsterme) scheint mir eine Bijektion zu
bestehen.
Von der unpassenden Verwendung des Pluszeichens mal abgesehen, dürfte
das so nicht hinkommen, weil man wohl nicht für jede Funktion einen
eindeutigen Funktionsterm finden dürfte.

Ich befürchte nicht nur, dass Hans' Ausführungen spurloa an Jürgen
vorübergegangen sind, sondern vielmehr, dass jener immer noch nicht
weiß, was eine Funktion überhaupt ist.

hs
IV
2018-04-23 19:01:25 UTC
Permalink
Post by IV
Post by Hans Crauel
Post by IV
Funktionen sind durch ihren Definitionsbereich und ihre
Zuordnungsvorschrift mit dem Funktionsterm beschrieben. Zwischen
Funktion und (DB + Funktionsvariable + Funktionsterm) scheint mir eine
Bijektion zu bestehen.
Nein. Im allgemeinen gibt es keinen "Funktionsterm".
Da ich in meinem Projekt die Funktionen über ihren Funktionsterm
handhaben muß, ...
Beziehen sich dann also die Sätze, die dieses "Projekt" hervorbringen wird
auf "Funktionen, die mittels Funktionsterm beschrieben werden"?
Oder wie muss man sich das vorstellen?
Ist das immer noch nicht deutlich geworden? Anders hätten Liouville, Ritt
und Rosenlicht ihre Funktionenklassen weder definieren noch behandeln
können.
Post by IV
Zwischen Funktion und (DB + Funktionsvariable + Funktionsterme) scheint
mir eine Bijektion zu bestehen.
Von der unpassenden Verwendung des Pluszeichens mal abgesehen, dürfte das
so nicht hinkommen, weil man wohl nicht für jede Funktion einen
eindeutigen Funktionsterm finden dürfte.
Dawegen schrub ich jetzt "Funktionsterme". Zwischen den Funktionstermen ein
und derselben nicht stückweise definierten Funktion scheinen mir paarweise
Bijektionen zu bestehen.
H0Iger SchuIz
2018-04-24 06:02:05 UTC
Permalink
Post by IV
Dawegen schrub ich jetzt "Funktionsterme".
Schrubb mal fleißig weiter, vielleicht hilft's.
Post by IV
Zwischen den Funktionstermen ein
und derselben nicht stückweise definierten Funktion scheinen mir paarweise
Bijektionen zu bestehen.
Und bei der Wortfolge bin ich mir noch nicht mal sicher, ob sie in der
Intention zusammengestellt wurde, Sinn zu ergeben, oder ob hier nur
vermeintlich mathematisch klingende Begriffe genannt werden sollten.

For starters: Was sind "paarweise Bijektionen"?
IV
2018-04-24 17:52:42 UTC
Permalink
Post by H0Iger SchuIz
Zwischen den Funktionstermen ein und derselben nicht stückweise
definierten Funktion scheinen mir paarweise Bijektionen zu bestehen.
Was sind "paarweise Bijektionen"?
Versuch's doch ausnahmsweise mal mit einer Interpretation die einen Sinn
ergeben könnte.
Bijektionen, die paarweise zwischen den Funktionstermen bestehen.
H0Iger SchuIz
2018-04-24 18:40:09 UTC
Permalink
Post by IV
Post by H0Iger SchuIz
Zwischen den Funktionstermen ein und derselben nicht stückweise
definierten Funktion scheinen mir paarweise Bijektionen zu bestehen.
Was sind "paarweise Bijektionen"?
Versuch's doch ausnahmsweise mal mit einer Interpretation die einen Sinn
ergeben könnte.
Bijektionen, die paarweise zwischen den Funktionstermen bestehen.
Das soll Sinn ergeben? Na, dann.

hs
Martin Vaeth
2018-04-24 06:07:02 UTC
Permalink
"H0Iger SchuIz" schrieb im Newsbeitrag
Beziehen sich dann also die Sätze, die dieses "Projekt" hervorbringen wird
auf "Funktionen, die mittels Funktionsterm beschrieben werden"?
Oder wie muss man sich das vorstellen?
Ist das immer noch nicht deutlich geworden? Anders hätten Liouville, Ritt
und Rosenlicht ihre Funktionenklassen weder definieren noch behandeln
können.
Zumindest Ritt und Rosenlicht sprechen von analytischen Funktionen
(Liouvilles Arbeiten dazu kenne ich nicht), wobei sie implizit wohl
Funktionen identifizieren, die auf irgendeiner nichtleeren offenen
Menge den selben Funktionszweig haben (können?). Zu ihren Zeiten war
das wohl noch der gängige Funktionsbegriff (aus dieser Historie
kommt ja der Begriff "Funktionentheorie", der für unser heutiges
Funktionsverständnis eine krasse Fehlbezeichnung ist), und
deswegen lassen sie sich darüber auch gar nicht aus.

Die Eigenschaft "elementar" zu sein (also durch einen "Funktionsterm"
beschrieben zu sein) ist eine spezielle *Eigenschaft* solcher Funktionen
bzw. Äquivalenzklasen. Das Objekt der Untersuchung sind aber eben die
genannten Funktionen/Äquivalenzklassen.
Zwischen den Funktionstermen ein
und derselben nicht stückweise definierten Funktion scheinen mir paarweise
Bijektionen zu bestehen.
Nein. Beispielsweise sind z^2 + z und z(z+1) oder exp(2 log z) + log(exp z)
oder log(exp z)(exp(log z) + 1) verschiedene Funktionsterme der gleichen
Funktion (bzw. Äquivalenzklassen) im Sinne von Ritt und Rosenlicht.
Diese Nichteindeutigkeit ist gerade das Problem von Ritts - wie Du ihn
nennst - "Struktursatz": Auch die Darstellung im Sinne von diesem
"Struktursatz" ist nicht eindeutig, da die selbe Funktion etwa einmal
als z und einmal als log((exp z^1)^1) geschrieben werden kann, und
beide Darstellungen die von Ritt beschriebenen Gestalt haben.
Genau deswegen ist es mit diesem Satz alleine eben nicht so trivial zu
zeigen, dass die Umkehrfunktion von z exp z nicht elementar ist,
da die "Kandidaten" für die Umehrfunktion gemäß Ritts Struktursatz
eben noch bei weitem nicht eindeutig charakterisiert sind.
IV
2018-04-24 19:23:42 UTC
Permalink
Post by Martin Vaeth
Post by IV
Post by H0Iger SchuIz
Beziehen sich dann also die Sätze, die dieses "Projekt" hervorbringen
wird auf "Funktionen, die mittels Funktionsterm beschrieben werden"?
Oder wie muss man sich das vorstellen?
Ist das immer noch nicht deutlich geworden? Anders hätten Liouville, Ritt
und Rosenlicht ihre Funktionenklassen weder definieren noch behandeln
können.
Zumindest Ritt und Rosenlicht sprechen von analytischen Funktionen
(Liouvilles Arbeiten dazu kenne ich nicht), wobei sie implizit wohl
Funktionen identifizieren, die auf irgendeiner nichtleeren offenen Menge
den selben Funktionszweig haben (können?). Zu ihren Zeiten war das wohl
noch der gängige Funktionsbegriff (aus dieser Historie kommt ja der
Begriff "Funktionentheorie", der für unser heutiges Funktionsverständnis
eine krasse Fehlbezeichnung ist), und deswegen lassen sie sich darüber
auch gar nicht aus.
Die Eigenschaft "elementar" zu sein (also durch einen "Funktionsterm"
beschrieben zu sein) ist eine spezielle *Eigenschaft* solcher Funktionen
bzw. Äquivalenzklassen. Das Objekt der Untersuchung sind aber eben die
genannten Funktionen/Äquivalenzklassen.
Liouville nennt seine Theorie "Integration in endlichen Termen", das heißt,
in geschlossenen Ausdrücken. Durch sukzessives Anwenden einer endlichen
Anzahl von algebraischen Operationen (= algebraischen Funktionen), exp
und/oder ln wird die Funktionenklasse "Elementare Funktionen" erzeugt. Durch
Hinzunahme der Integration wird die Funktionenklasse "Liouvillesche
Funktionen" erzeugt. Heute verwendet man die Begriffe "Körper" und
"Differentialkörper". Jeder auf solche systematische Weise erzeugten
Funktion lassen sich Funktionsterme zuordnen. Erst durch solche
systematischen Aufbauprinzipien lassen sich derartige Funktionenklassen
behandeln.
Post by Martin Vaeth
Post by IV
Zwischen den Funktionstermen ein und derselben nicht stückweise
definierten Funktion scheinen mir paarweise Bijektionen zu bestehen.
Nein. Beispielsweise sind z^2 + z und z(z+1) oder exp(2 log z) + log(exp
z) oder log(exp z)(exp(log z) + 1) verschiedene Funktionsterme der
gleichen Funktion (bzw. Äquivalenzklassen) im Sinne von Ritt und
Rosenlicht.
Wie Du richtig feststellst, sind das alles, über dem selben
Definitionsbereich, Funktionsterme ein und derselben Funktion. Und zwischen
jeweils zweien dieser Funktionsterme besteht eine Bijektion. Warum also Dein
"Nein"?
Post by Martin Vaeth
Diese Nichteindeutigkeit ist gerade das Problem von Ritts - wie Du ihn
nennst - "Struktursatz": Auch die Darstellung im Sinne von diesem
"Struktursatz" ist nicht eindeutig, da die selbe Funktion etwa einmal als
z und einmal als log((exp z^1)^1) geschrieben werden kann, und beide
Darstellungen die von Ritt beschriebene Gestalt haben.
Man sollte die Dinge nicht nur von einer Seite aus betrachten. Ritts Satz
sagt nicht, welcherart bijektive elementare Funktionen elementar umkehrbar
sind. Da hast Du recht. Er sagt, welcherart bijektive elementare Funktionen
n i c h t elementar umkehrbar sind.
Post by Martin Vaeth
Diese Nichteindeutigkeit ist gerade das Problem von Ritts - wie Du ihn
nennst - "Struktursatz": Auch die Darstellung im Sinne von diesem
"Struktursatz" ist nicht eindeutig, da die selbe Funktion etwa einmal als
z und einmal als log((exp z^1)^1) geschrieben werden kann, und beide
Darstellungen die von Ritt beschriebene Gestalt haben.
Genau deswegen ist es mit diesem Satz alleine eben nicht so trivial zu
zeigen, dass die Umkehrfunktion von z exp z nicht elementar ist, da die
"Kandidaten" für die Umehrfunktion gemäß Ritts Struktursatz eben noch bei
weitem nicht eindeutig charakterisiert sind.
Niemand behauptet etwas Anderes. Ich möchte doch nur Ritts Satz auf andere
Funktionenklassen erweitern - mehr erstmal nicht.
Später dann vielleicht könnte man in einem anderen Projekt schauen, ob sich
durch meine Übersetzung in den treffenden Begriff "algebraische
Unabhängigkeit" genügend leistungsfähige Methoden der Transzendenten
Zahlentheorie finden lassen.
Ihr wollt hier immer, daß ich den zweiten Schritt vor dem ersten mache,
obwohl ich noch gar nicht soweit bin. Also gut: Wie kann man zeigen, daß
eine Funktion F: Z offen \neq \emptyset \subseteq \mathbb{C} --->
\mathbb{C}, F = id o exp, keine Darstellung F = A o f haben kann, in der A
eine einstellige algebraische Funktion und f eine transzendente Funktion
ist?
Mir scheint, aus meinem Satz unten aus news:pad5a1$7la$***@news.albasani.net
ergibt sich folgendes.
Wenn F eine Funktion ist mit F: Z Gebiet \subseteq \mathbb{C} \to
\mathbb{C}, F(z) = A( f1(z), f2(z)), worin A eine zweistellige algebraische
Funktion ist und f1 und f2 Funktionen f1 und f2 aus dem Satz unten sind,
dann gibt es für F keine Darstellung A1 o f, worin A1 eine einstellige
algebraische Funktion ist und f eine transzendente elementare Funktion.

Satz:
Seien f1, f2, g holomorphe Funktionen mit
f1: Z1 Gebiet \subseteq \mathbb{C} \to \mathbb{C},
f2: Z2 Gebiet \subseteq \mathbb{C} \to \mathbb{C},
g: Z3 Gebiet \subseteq \mathbb{C} \to \mathbb{C}, g nicht konstant,
und sei Z1 \cap Z2 \cap g(Z3) \neq \emptyset.
Wenn f1 und f2 über dem Koeffizientenkörper \mathbb{C} algebraisch
unabhängig sind, dann sind die Funktionen F1: Z \to \mathbb{C}, z \mapsto
f1(g(z)) und F2: Z \to \mathbb{C}, z \mapsto f2(g(z)) über dem
Koeffizientenkörper \mathbb{C} algebraisch unabhängig.


Übrigens sitze ich jetzt am Formulieren des Beweises von Ritts erstem
wirklichen Satz Lemma 18 im allgemeinen Fall, nämlich nicht nur für
Elementare Funktionen, sondern für n mal differenzierbare Funktionen (mit
Komposition als Aufbauprinzip).
Es ist doch alles sehr viel einfacher als ich bisher dachte, denn Ritt
schreibt unnötig umständlich.
IV
2018-04-24 19:43:10 UTC
Permalink
Post by Martin Vaeth
Zwischen den Funktionstermen ein und derselben nicht stückweise
definierten Funktion scheinen mir paarweise Bijektionen zu bestehen.
Nein. Beispielsweise sind z^2 + z und z(z+1) oder exp(2 log z) + log(exp
z) oder log(exp z)(exp(log z) + 1) verschiedene Funktionsterme der
gleichen Funktion (bzw. Äquivalenzklassen) im Sinne von Ritt und
Rosenlicht.
Ich glaube jetzt hab' ich's: Bijektion ist der falsche Begriff. In der Menge
{Funktion F, {t | t ist Funktionsterm von F}} besteht eine
Äquivalenzrelation.
H0Iger SchuIz
2018-04-25 06:24:32 UTC
Permalink
Post by IV
Ich glaube jetzt hab' ich's: Bijektion ist der falsche Begriff.
Aha.
Post by IV
In der
Menge {Funktion F, {t | t ist Funktionsterm von F}}
Diese Menge hat zwei Elemente. Womöglich stolpert hier der Amateur aber
über die Schreibweisen.
Post by IV
besteht eine
Äquivalenzrelation.
Das soll eine Erkenntnis sein? Man kann auf jeder Menge eine
Äquivalenzrelation definieren.

hs
IV
2018-04-25 15:08:14 UTC
Permalink
Post by H0Iger SchuIz
In der Menge {Funktion F, {t | t ist Funktionsterm von F}}
Diese Menge hat zwei Elemente. Womöglich stolpert hier der Amateur aber
über die Schreibweisen.
Ja. Ich hab' wieder zu schnell geschrieben. Es ist aber trotzdem richtig.
Man kann auch formulieren: {Funktion F} \cup {t | t ist Funktionsterm von
F}}.
Post by H0Iger SchuIz
besteht eine Äquivalenzrelation.
Das soll eine Erkenntnis sein? Man kann auf jeder Menge eine
Äquivalenzrelation definieren.
Es ist keine Erkenntnis. Ihr wolltet mir nur wieder mal nicht glauben, daß
sowohl jeder der Funktionsterme als auch der Funktionsbezeichner, über
demselben Definitionsbereich, ein Repräsentant für die Funktion ist.
IV
2018-04-24 19:47:56 UTC
Permalink
"IV" schrieb im Newsbeitrag news:pbo082$sv3$***@news.albasani.net...
Ich hab' doch wieder was vergessen hinzuschreiben:
In "keine Darstellung F = A o f haben kann, in der A eine einstellige
algebraische Funktion und f eine transzendente Funktion ist?" muß korrigiert
werden: "und f eine transzendente elementare Funktion".
H0Iger SchuIz
2018-04-25 06:24:32 UTC
Permalink
Post by IV
Funktion F: Z offen \neq \emptyset \subseteq \mathbb{C} --->
\mathbb{C},
Hier noch eine Anmerkung zu den Schreibweisen. Es mag zwar besonders
lässig wirken, die Eigenschaften der Definitionsmenge quasi als
Nebensatz unterzumischen. Formal sauber wird das aber nicht. Im
vorliegenden Fall hat man dann zwar aufgeschrieben, dass die leere Menge
eine Teilmenge der Menge der komplexen Zahlen ist, nicht jedoch dass der
Definitionsbereich $Z$ eine Teilmenge der Menge der komplexen Zahlen
ist.

hs
H0Iger SchuIz
2018-04-25 06:24:32 UTC
Permalink
Post by IV
Wie kann man zeigen, daß
eine Funktion F: Z offen \neq \emptyset \subseteq \mathbb{C} --->
\mathbb{C}, F = id o exp, keine Darstellung F = A o f haben kann, in der A
eine einstellige algebraische Funktion und f eine transzendente Funktion
ist?
Ist es nicht so, das F = id o exp daselbst schon eine Darstellung als
Kompositions aus einer algebraischen und einer transzendenten Funktion
ist? Dann dürfte es eher schwierig werden, zu zeigen, dass eine solche
Darstellung nicht gibt.

hs
IV
2018-04-25 15:38:01 UTC
Permalink
Post by H0Iger SchuIz
Wie kann man zeigen, daß eine Funktion F: Z offen \neq \emptyset
\subseteq \mathbb{C} ---> \mathbb{C}, F = id o exp, keine Darstellung F =
A o f haben kann, in der A eine einstellige algebraische Funktion und f
eine transzendente Funktion ist?
Ist es nicht so, das F = id o exp daselbst schon eine Darstellung als
Kompositions aus einer algebraischen und einer transzendenten Funktion
ist? Dann dürfte es eher schwierig werden, zu zeigen, dass eine solche
Darstellung nicht gibt.
Oh, danke. Ja, es ist so.
Ich sollte doch lieber zu der vertrauteren Funktionstermschreibweise
zurückkehren.
Gemeint sind die Funktionen F mit dem Funktionsterm F(z) = z * exp(z).
Martin Vaeth
2018-04-25 10:48:24 UTC
Permalink
Post by IV
Post by Martin Vaeth
Zwischen den Funktionstermen ein und derselben nicht stückweise
definierten Funktion scheinen mir paarweise Bijektionen zu bestehen.
Nein. Beispielsweise sind z^2 + z und z(z+1) oder exp(2 log z) + log(exp
z) oder log(exp z)(exp(log z) + 1) verschiedene Funktionsterme der
gleichen Funktion (bzw. Äquivalenzklassen) im Sinne von Ritt und
Rosenlicht.
Wie Du richtig feststellst, sind das alles, über dem selben
Definitionsbereich, Funktionsterme ein und derselben Funktion.
Nicht ganz. Der Definitionsbereich des Logarithmus ist nicht \C
(weswegen ich nochmals den Begriff der "Äquivalenzklasse" betonte,
mit denen Ritt wohl arbeitet, ohne es zu spezifizieren), aber das
ist für unsere jetzige Diskussion nicht wesentlich.
Post by IV
Und zwischen
jeweils zweien dieser Funktionsterme besteht eine Bijektion.
Dieser Satz ergibt keinen Sinn für mich.
Post by IV
Warum also Dein "Nein"?
Ich habe Deine Aussage als eine verknappte Formulierung verstanden
für die Aussage: "Es besteht eine (natürliche) paarweise Zuordnung
zwischen einem Funktionsterm und der zugehörigen Funktion;
also eine Bijektion von der Menge der Funktionsterme auf die
Menge der (elementaren) Funktionen."
Auf die letzte Aussage bezog sich meine Antwort (in der ich zeige, dass
diese vermeintliche Bijektion nicht injektiv ist; daher "Nein").
Post by IV
Post by Martin Vaeth
z und einmal als log((exp z^1)^1) geschrieben werden kann, und beide
Darstellungen die von Ritt beschriebene Gestalt haben.
Man sollte die Dinge nicht nur von einer Seite aus betrachten. Ritts Satz
sagt nicht, welcherart bijektive elementare Funktionen elementar umkehrbar
sind. Da hast Du recht. Er sagt, welcherart bijektive elementare Funktionen
n i c h t elementar umkehrbar sind.
Wenn der Satz die zweite Klasse von Funktionen charakterisiert,
charakterisiert er auch die erste (als deren Komplement).

Ich glaube, Du hast den Zweck meines Beispiels missverstanden:
Mit diesem Beispiel wollte ich nicht Ritts Satz anzweifeln,
sondern einfach nur zeigen, dass die "Struktur" aus diese Satz
(Komposition von abwechselnd Polynomen und Exp/Log zu sein),
nicht sehr stark ist und eben selbst im trivialsten Fall der
Identitätsfunktion keine Eindeutigkeit sichert.
Ganz anders sieht es aus bei Rosenblums/Rischs "Struktursatz"
der elementar integrierbaren Funktiobnen, wo es um *Summen*
solcher Terme geht und man daher zumindest im Falle algebraischer
Unabhängeitkeit eine gewisse Eindeutigkeit der Darstellung hat,
die sich dann beweisbar (und sogar algorithmisch) ermitteln lässt.
Post by IV
Ihr wollt hier immer, daß ich den zweiten Schritt vor dem ersten mache,
obwohl ich noch gar nicht soweit bin.
Ich will gar nichts von Dir. Aber ich halte aus den genannten Gründen
Ritts "Struktursatz" bei weitem nicht so nützlich wie Du.
Aber jetzt bin ich still, denn ich möchte Dein Vorhaben nicht
schlechtreden: Invertierbarkeit in einer Funktionenklasse *ist*
halt vermutlich ein schwierigeres Problem als Integrierbarkeit.
H0Iger SchuIz
2018-04-25 14:27:53 UTC
Permalink
Post by Martin Vaeth
"Es besteht eine (natürliche) paarweise Zuordnung
zwischen einem Funktionsterm und der zugehörigen Funktion;
also eine Bijektion von der Menge der Funktionsterme auf die
Menge der (elementaren) Funktionen."
Richtig klar wird's nicht, was er meint. Von der Idee, die Funktionen
(eindeutig) mit Funktionstermen identifizieren zu können, scheint er
sich ja gelöst zu haben.

Mit den "Bijektionen" oder "Äquivalenzrelationen" will er vielleicht
ausdrücken, dass (syntaktisch) unterschiedliche Funktionsterme der
gleichen Funktion an den gleichen Stellen auch gleiche Werte liefern.
Das ist baer mehr als trivial und keiner Beachtung wert.

Das sind aber alles nur Vermutung, so wie er es schreibt, macht's
einfach keinen Sinn. Die Aufforderung, es doch einfach sinnvoll zu
interpretiere´n, entspricht einer, mit einem Stück nasser Seife ein
Streichholz anzuzünden -- no chance at all.

hs
IV
2018-04-25 15:18:51 UTC
Permalink
Post by IV
Post by Martin Vaeth
Diese Nichteindeutigkeit ist gerade das Problem von Ritts - wie Du ihn
nennst - "Struktursatz": Auch die Darstellung im Sinne von diesem
"Struktursatz" ist nicht eindeutig, da die selbe Funktion etwa einmal
als z und einmal als log((exp z^1)^1) geschrieben werden kann, und beide
Darstellungen die von Ritt beschriebene Gestalt haben.
Man sollte die Dinge nicht nur von einer Seite aus betrachten. Ritts Satz
sagt nicht, ...
Oh, entschuldigt. Ich habe wieder schneller geschrieben als nachgedacht.
Also: Aus Ritts Satz ergibt sich, daß die Umkehrfunktion einer bijektiven
elementaren Funktion die einen Funktionsterm in
expln-Kompositionsdarstellung hat in dem keine mehrstellige algebraische
Funktion vorkommt, eine elementare Funktion ist. Das ist doch ein sehr
nützliches Instrument für die Praxis.
IV
2018-04-25 15:33:41 UTC
Permalink
Auch die Darstellung im Sinne von diesem "Struktursatz" ist nicht
eindeutig, da die selbe Funktion etwa einmal als z und einmal als log((exp
z^1)^1) geschrieben werden kann, und beide Darstellungen die von Ritt
beschriebenen Gestalt haben.
Selbstverständlich kann man jeden Funktionsterm komplizierter formulieren.
Aber aus Ritts Satz ergibt sich, daß es einen Unterschied gibt zwischen
Funktionstermen in expln-Kompositionsdarstellung mit und ohne mehrstellige
algebraische Funktionen. Im übrigen gehören die beiden von Dir genannten
Funktionen letzterem Typ an. Da sie (über nichtleerem offenem
Definitionsbereich) bijektiv ist, ergibt sich aus Ritts Satz sofort, daß die
Umkehrfunktion eine elementare Funktion ist.
Der von Dir hier in diesem Beispiel angesprochenen Art von
Nichteindeutigkeit begegnet Ritt damit, daß er fordert, man solle die
Funktionsterme mit der geringsten Anzahl von algebraisch unabhängigen
Monomen als Argumente der äußersten algebraischen Funktion betrachten. Ich
vermute, die minimale Anzahl solcher Monome könnte eine Invariante der
Funktion sein.
IV
2018-04-20 15:30:55 UTC
Permalink
Post by Detlef Müller
Post by IV
Post by Torn Rumero DeBrak
Post by IV
Beispiel exp und ln.
Also ist exp(z) nur ein anderer Name für den Term
sum_i=0^oo (z^i / i! ),
und ln(z) nur eine andere, verkürzte Bezeichnung für den Term
sum_i=1^oo ( (-1)^(i-1) * (z-1)^i / i )
richtig?
Ich denke mal, ja.
Sind dann die Terme
sum_i=0^oo (z^i / i! )
und
lim_{n->oo}(1+z/n)^n
algebraisch unabhängig?
(Ich weiß wieder mal gar nicht, ob das wieder eine Hausaufgabe, eine
Testfrage oder eine echte Frage sein soll.)
Ich habe hier bei Euch gelernt, daß zu einem Term immer auch sein
Definitionsbereich gehört. Das gilt selbst für die Termumformung und für die
Termvereinfachung.
Ich denke mal, wenn die beiden Terme oben über dem gesamten gegebenen
Definitionsbereich Funktionsterme der beiden Funktionen sind, dann sind auch
diese beiden Terme über diesem Definitionsbereich algebraisch unabhängig.

Die Abhängigkeit zwischen Funktionen wird doch durch Verschwinden der
Funktionaldeterminante (Jacobi-Determinante) gezeigt. Zeigt man da
vielleicht tatsächlich nur die algebraische Abhängigkeit? Damit könntet Ihr
dann doch Detlefs obige Frage beantworten.
Hans Crauel
2018-04-21 21:03:40 UTC
Permalink
IV schrieb
"Detlef MÃŒller" schrieb
Post by Detlef Müller
Sind dann die Terme
sum_i=0^oo (z^i / i! )
und
lim_{n->oo}(1+z/n)^n
algebraisch unabhÀngig?
(Ich weiß wieder mal gar nicht, ob das wieder eine Hausaufgabe, eine
Testfrage oder eine echte Frage sein soll.)
Es handelt sich um eine echte Frage nach dem von dir verwendeten
Begriff von "algebraisch unabhaengig". Diesen Begriff musst schon
du genau spezifizieren. Ansonsten ist es nicht moeglich, Fragen
nach "algebraischer Unabhaengigkeit" sinnvoll zu beantworten. Das
gibt nur Rumstocherei und Geschwafel.
Ich habe hier bei Euch gelernt, daß zu einem Term immer auch sein
Definitionsbereich gehört.
Dann hast du etwas Falsches gelernt. Ein Term ist keine Funktion.
Eine Funktion kann mittels eines Terms definiert werden, wenngleich
nicht fuer Werte der Variablen, fuer welche der Term unsinnig ist.
Etwa 1/(1-x) ist keine Funktion, sondern ein Term, der fuer x = 1
sinnlos ist. Durch diesen Term laesst sich allerdings z.B. eine
Funktion von (1,infty) nach R definieren (die ist nach oben durch
0 beschraenkt), oder eine von [0,1) nach R (die ist dann nicht
nach oben beschraenkt), oder eine von (-infty,0] cup [2,infty)
nach R (die ist dann insgesamt beschraenkt), oder diverse andere.
Ich denke mal, wenn die beiden Terme oben ÃŒber dem gesamten gegebenen
Definitionsbereich Funktionsterme der beiden Funktionen sind, dann sind auch
diese beiden Terme Ìber diesem Definitionsbereich algebraisch unabhÀngig.
Die beiden Terme, die Detlef hingeschrieben hat, sind (nicht nur
notationell) verschieden, sie sind aber fuer alle z aus C sinnvoll,
und nimmt man die durch die beiden Terme definierten Funktionen
auf C, so stellt man fest, dass sie gleich sind.
Es ist schon ein arg eigenartiger Begriff von "algebraischer
Unabhaengigkeit", wenn diese beiden Funktionen algebraisch
unabhaengig sein sollen.

Hans
IV
2018-04-22 19:34:35 UTC
Permalink
Post by Detlef Müller
Sind dann die Terme
sum_i=0^oo (z^i / i! )
und
lim_{n->oo}(1+z/n)^n
algebraisch unabhängig?
(Ich weiß wieder mal gar nicht, ob das wieder eine Hausaufgabe, eine
Testfrage oder eine echte Frage sein soll.)
Es handelt sich um eine echte Frage nach dem von dir verwendeten Begriff
von "algebraisch unabhaengig". Diesen Begriff musst schon du genau
spezifizieren. Ansonsten ist es nicht moeglich, Fragen nach "algebraischer
Unabhaengigkeit" sinnvoll zu beantworten.
In der mathematischen Literatur gibt es Definitionen des Begriffs "(über
einem Körper) algebraisch unabhängige Funktionen". Ich hatte gehofft, daß
Ihr mir da weiterhelfen könnt.
Ich habe hier bei Euch gelernt, daß zu einem Term immer auch sein
Definitionsbereich gehört.
Dann hast du etwas Falsches gelernt. Ein Term ist keine Funktion.
Wenn man aber Termumformung betreiben möchte, dann muß man die Gültigkeits-
und Wertebereich der Terme berücksichtigen. Ein Term scheint mir daher eine
Relation zu repräsentieren.
Ich denke mal, wenn die beiden Terme oben über dem gesamten gegebenen
Definitionsbereich Funktionsterme der beiden Funktionen sind, dann sind
auch diese beiden Terme über diesem Definitionsbereich algebraisch
unabhängig.
Die beiden Terme, die Detlef hingeschrieben hat, sind (nicht nur
notationell) verschieden, sie sind aber fuer alle z aus C sinnvoll, und
nimmt man die durch die beiden Terme definierten Funktionen auf C, so
stellt man fest, dass sie gleich sind.
Es ist schon ein arg eigenartiger Begriff von "algebraischer
Unabhaengigkeit", wenn diese beiden Funktionen algebraisch unabhaengig
sein sollen.
(Ich war davon ausgegangen, daß die beiden Terme die beiden genannten
Funktionen exp und ln repräsentieren. Deshalb hatte ich oben auch von "den
beiden Funktionen" gesprochen. Jetzt habe ich mir Detlefs zwei Terme mal
aufgeschrieben. Es bleibt wie oben von mir geschrieben: Wenn Detlefs beide
Terme die beiden algerbaisch unabhängigen Funktion exp und ln
repräsentieren, dann sind sie natürlich ebenfalls algebraisch unabhängig.
Wenn sie aber beide dieselbe Funktion, exp, über deren gesamten
Definitionsbereich repräsentieren, sind sie selbstverständlich algebraisch
abhängig.)
Torn Rumero DeBrak
2018-04-13 17:33:32 UTC
Permalink
Post by IV
Post by Torn Rumero DeBrak
Post by IV
Post by Torn Rumero DeBrak
Post by IV
Post by IV
Post by IV
Algebraische Unabhängigkeit
Und ist meine Definition auch richtig? Die ist für mich
(jedenfalls im Moment noch) wesentlich anschaulicher.
Die andere Richtung könnte haariger zu zeigen sein ... vielleicht
mit dem Satz über implizite Funktionen?
Womöglich müssen dann gewisse Punkte aus dem Defbereich genommen
werden.
Ich sehe das folgendermaßen. Ich will doch allgemeine Aussagen für
Funktionenklassen herleiten. Da kann ich doch nicht das
Wachstumsverhalten jeder einzelnen Funktion betrachten. Deshalb
mußte ich die algebraische Abhängigkeit der Funktionsterme mit der
der Funktionswerte in Zusammenhang bringen.
Da bezweifle ich aber, dass man aus diesem Zusammenhang etwas machen kann.
z. B. gibt es algebraisch unabhängige Funktionen, deren
Funktionswerte an derselben Stelle aber algebraisch abhängig sind.
Schau dir nur exp(z) und ln(z+1) an der Stelle z=0 an.
(f1, f2 algebraisch unabhängig => f1(z_0) und f2(z_0) algebraisch
unabhängig)
(f1(z_0) und f2(z_0) algebraisch abhängig => f1, f2 algebraisch abhängig)
Das oben sind die Aussagen, die durch mein Beispiel belegt werden.
Deshalb folgt nicht aus der algebraischen Unabhängigkeit der Funktionen
die alg. Unabh. der Funktionswerte an einer Stelle oder umgekehrt.
Post by IV
Post by Torn Rumero DeBrak
Post by IV
Das haben die forschenden Mathematiker in MathOverflow
https://mathoverflow.net/questions/296676/algebraic-independence-of-the-composition-of-functions
doch schon beantwortet: Die Funktionen dürfen in der Schnittmenge
ihrer Definitionsbereiche nicht konstant sein.
Was hat das mit meiner Aussage zu tun? Ich habe keine konstante
Funktionen erwähnt. Was interpretierst du immer in das geschriebene
Wort hinein?
Du sprichst von der Stelle z=0, und z_0 interpretiere ich auch als
Stelle, nicht als Variable.
Soweit, so gut.
Post by IV
In \mathbb{C} dürften die algebraischen Zahlen isoliert liegen, und
damit für holomorphe transzendente Funktionen über einem Gebiet auch die
"exceptional points".
Und mehr noch: f1 und f2 algebraisch unabhängig heißt, das gilt für
f1(Z) und f2(Z) für alle z aus der Schnittmenge Z der beiden
Definitionsbereiche.
Das bezweifle ich stark, wenn du keinen Beweis hast. Oben hatte ich doch
ein Gegenbeispiel zu dieser deiner Vermutung gezeigt.
Wenn dier Funktiunen algebraisch unabhängig sind, so sind es NICHT auch
ihre Funktionswerte an einer Stelle.

Ich lasse mich aber durch einen Beweis überzeugen.
Post by IV
In jeder Umgebung zu einem algebraisch abhängigen
Funktionswert liegt immer auch ein algebraisch unabhängiger
Funktionswert, weshalb immer auch f1(Z) und f2(Z) algebraisch unabhängig
sind - außer bei "irgendwo" in Z konstanten Funktionen. Darauf weist der
Kommentar in MathOverflow hin, und darauf, daß nichtkonstante holomorphe
Funktionen über einem Gebiet "nirgendwo" konstant sind (nicht stückweise
konstant, nicht lokal konstant und nicht konstant - IV). Aber wegen
meines Beweises des Satzes über die algebraische Unabhängigkeit ganz
oben vom Anfang dieser Diskussion hier will ich demnächst mal hier
nachfragen.
Das war nicht mein Anliegen in der Antwort zu deiner falsche Vermutung.
Post by IV
Post by Torn Rumero DeBrak
Deshalb noch einmal meine Frage: Weißt du, wann zwei Funktionen gleich
sind?
Ja. Und wir hatten das hier schon zwei, drei Mal aufgeschrieben.
Post by Torn Rumero DeBrak
Ich glaube nämlich, du weißt es nicht, so wie du immer am Thema vorbei
antwortest.
Willst Du mich denn auf einen konkreten Fehler bezüglich der Anwendung
dieser Definition hinweisen?
Post by Torn Rumero DeBrak
Ich bezog mich auf deine Aussage "Deshalb mußte ich die algebraische
Abhängigkeit der Funktionsterme mit der der Funktionswerte in
Zusammenhang bringen."
Machst du das nun oder machst du das nicht, wenn MathOverflow dir
davon abrät?
Ich weiß nicht, warum Du an dieser Stelle MathOverflow ins Spiel
bringst.
DU HATTEST MathOverflow als Link ins Spiel gebracht.
Post by IV
In meinem Link wurde doch vom Beweis, den man "can easily see"
gar nichts preisgegeben.
Wie oben in diesem Post muß ich wahrscheinlich anstelle von f1(z)  und
f1 und f2 algebraisch unabhängig <=> f1(Z) und f2(Z) algebraisch unabhängig.
DAS IST MEINER MEINUNG NACH EINE FALSCHE AUSSAGE. (unabhängig, ob du z
groß oder klein schreibst).
Post by IV
Oder meinst Du etwas Anderes?
Um diese Aussage scheint es dir doch zu gehen, oder etwa nicht?
Post by IV
(Wenn Du das klar benennst geht alles viel schneller, brauche ich nicht
rumrätseln, Euch nicht langweilen und Euch nicht nerven.)
Was soll ich sonst noch sagen: Du verstehst einfach nicht, was ich
schreibe. Mach es einfach, du verkomplizierst alles unnötig.
Fange erst einmal klein an.

Deshalb meine Frage:

Weisst du, wann zwei Funktionen gleich sind? Bitte nicht nur mit ja
antworten, denn ich möchte wissen, ob du da einem Irrtum erliegst,
den man möglicherweise korrigieren kann.

Und die nächst Frage wäre dann:

Weisst du, wann zwei Polynome mit Koeffizienten in einem Körper gleich
sind?

Aloha
IV
2018-04-13 21:35:32 UTC
Permalink
Post by Torn Rumero DeBrak
Post by IV
Post by Torn Rumero DeBrak
Post by Torn Rumero DeBrak
(f1, f2 algebraisch unabhängig => f1(z_0) und f2(z_0) algebraisch
unabhängig)
(f1(z_0) und f2(z_0) algebraisch abhängig => f1, f2 algebraisch abhängig)
Das oben sind die Aussagen, die durch mein Beispiel belegt werden.
Deshalb folgt nicht aus der algebraischen Unabhängigkeit der Funktionen
die alg. Unabh. der Funktionswerte an einer Stelle oder umgekehrt.
Das wollte ich nicht behauptet haben. f1(z) und f2(z) sind weder einzelne
Funktionswerte noch die Gesamtheit der Funktionswerte - es sind die
Funktionsterme.
Mit welchen Notationen kann man unterscheiden zwischen den Funktionswerten
f(z) und dem Funktionsterm f(z)?
Post by Torn Rumero DeBrak
Post by IV
Post by Torn Rumero DeBrak
Post by Torn Rumero DeBrak
Ich bezog mich auf deine Aussage "Deshalb mußte ich die algebraische
Abhängigkeit der Funktionsterme mit der der Funktionswerte in
Zusammenhang bringen."
Machst du das nun oder machst du das nicht, wenn MathOverflow dir davon
abrät?
Ich weiß nicht, warum Du an dieser Stelle MathOverflow ins Spiel bringst.
DU HATTEST MathOverflow als Link ins Spiel gebracht.
Ja, wo man mir durch kurzen Blick auf die Problemstellung bestätigt hat, daß
meine Vermutung mindestens für alle nichtkonstanten holomorphen Funktionen
über einem Gebiet zutrifft.
An welcher Stelle hat man mir wovon abgeraten? Es ging nur darum, daß die
Frage für MathOverflow zu einfach ist. Für MathStackExchange dagegen ist sie
zu allgemeingültig, und für hier scheint sie zu schwierig zu sein.
Post by Torn Rumero DeBrak
Weisst du, wann zwei Funktionen gleich sind?
Ja.
Post by Torn Rumero DeBrak
Weisst du, wann zwei Polynome mit Koeffizienten in einem Körper gleich
sind?
Ja.
Post by Torn Rumero DeBrak
Was soll ich sonst noch sagen: Du verstehst einfach nicht, was ich
schreibe. Mach es einfach, du verkomplizierst alles unnötig.
Nicht böse sein, das paßt hier nur gerade sprachlich gut:
Was soll ich sonst noch sagen: Du verstehst einfach nicht, was ich schreibe.
Mach es einfach, du verkomplizierst alles unnötig.
Das auf derselben Notation für zwei unterschiedliche Dinge (Funktionswerte
und Funktionsterm) beruhende Mißverständnis habe ich oben ausgeräumt. Damit
dürften sich jetzt Dein Einwand, Deine Kritik und Deine Testfragen erübrigt
haben, oder?
PeterSchneider
2018-04-14 09:37:26 UTC
Permalink
Post by Torn Rumero DeBrak
Weisst du, wann zwei Funktionen gleich sind?
Ja.
Post by Torn Rumero DeBrak
Weisst du, wann zwei Polynome mit Koeffizienten in einem Körper gleich
sind?
Ja.
Er hat gefragt:
Deshalb meine Frage:

Weisst du, wann zwei Funktionen gleich sind? Bitte nicht nur mit ja
antworten, denn ich möchte wissen, ob du da einem Irrtum erliegst,
den man möglicherweise korrigieren kann.
Und die nächst Frage wäre dann:

Weisst du, wann zwei Polynome mit Koeffizienten in einem Körper gleich
sind?
Dass du jetzt so ausweichst lässt tief blicken: du weißt es eben NICHT!
IV
2018-04-14 10:55:07 UTC
Permalink
Post by Torn Rumero DeBrak
Weisst du, wann zwei Funktionen gleich sind?
Ja.
https://de.wikibooks.org/wiki/Mathematik:_Analysis:_Grundlagen:_Funktionen
"Definition - Gleicheit von Funktionen"
Post by Torn Rumero DeBrak
Weisst du, wann zwei Polynome mit Koeffizienten in einem Körper gleich
sind?
Ja.
https://www.google.de/search?q=%22Gleichheit+von+Polynomen%22&sourceid=ie7&rls=com.microsoft:de-DE:IE-Address&ie=&oe=&gfe_rd=cr&dcr=0&ei=0tnRWrrcCYiDX5jYg6gM
Und wozu brauchen wir das jetzt hier? Wo ist ein Fehler in meiner
Argumentation/Notation? Sagt es doch bitte einfach.
(Im Moment sehe ich nur, daß Torn mein f1(z) als Funktionswert interpretiert
hat, nicht wie ich als Term einer Variablen z (Funktionsterm). Nur wenn ich
Torns Interpretation gemeint hätte, wäre deren Anwendung bei mir falsch und
hätte seinen Verweis auf die Gleichheit von Funktionen und Polynomen
gerechtfertigt.)
Ich hatte auch eine Menge Fragen gestellt.

Mit welcher Notation kann man unterscheiden zwischen einem Term f(z) der
Variablen z und Funktionswerten f(z)?
Torn Rumero DeBrak
2018-04-16 17:43:14 UTC
Permalink
Post by IV
Post by Torn Rumero DeBrak
Weisst du, wann zwei Funktionen gleich sind?
Ja.
https://de.wikibooks.org/wiki/Mathematik:_Analysis:_Grundlagen:_Funktionen
"Definition - Gleicheit von Funktionen"
Post by Torn Rumero DeBrak
Weisst du, wann zwei Polynome mit Koeffizienten in einem Körper
gleich sind?
Ja.
https://www.google.de/search?q=%22Gleichheit+von+Polynomen%22&sourceid=ie7&rls=com.microsoft:de-DE:IE-Address&ie=&oe=&gfe_rd=cr&dcr=0&ei=0tnRWrrcCYiDX5jYg6gM
Und wozu brauchen wir das jetzt hier? Wo ist ein Fehler in meiner
Argumentation/Notation? Sagt es doch bitte einfach.
(Im Moment sehe ich nur, daß Torn mein f1(z) als Funktionswert
interpretiert hat, nicht wie ich als Term einer Variablen z
(Funktionsterm). Nur wenn ich Torns Interpretation gemeint hätte, wäre
deren Anwendung bei mir falsch und hätte seinen Verweis auf die
Gleichheit von Funktionen und Polynomen gerechtfertigt.)
Ich hatte auch eine Menge Fragen gestellt.
Mit welcher Notation kann man unterscheiden zwischen einem Term f(z) der
Variablen z und Funktionswerten f(z)?
Ein Funktionsterm mit einem Bezeichner z ist eine Kombination aus
Konstanten, Rechenzeichen, Klammern zur Bestimmung der Abarbeitungsfolge
und dem lexikalische Ausdruck z, der rechts von f(z) = (Hier der Term)
steht, d.h. also ein Rechenausdruck ist (Beispiel: otto-anna/berta
mit den Bezeichnern otto, anna und berta).
Genaueres könnte eine Backus-Naur-Definition zeigen.

f(z) ist KEIN Term, da nichts über f(z) bekannt ist.
f(z) beschreibt hingegen üblicherweise den
Funktionswert der Funktion f an der Stelle (nicht Variablen) z. Zur
Verdeutlichung, dass es sich um eine Stelle aus dem Definitionsbereich
der Funktion handelt, wird z oft indiziert, also als z_0 dargestellt.

Eine Variable z hat nur in einer Gleichung Sinn, wo auf der rechten
Seite ein zu definierender Name mit der Variablen steht und links von
der Gleichung ein Term, der diese Variable enthält, z.B.

Rhabarber(f) := f*(a - 1/f)

wo in vielen Fällen ":=" statt "=" verwendet wird, um den wohl auch
von dir begangenen Fehler in der Interpretation von "="
zu verhindern.

Das geschieht dann so, dass die Gleichung unabhängig von der
gewählten Variablen ist. Man könnte also auch

Rhabarber(kompott) := kompott*(a - 1/kompott)

schreiben und erhält dasselbe Objekt. Wichtig ist nur, DIESELBE Variable
links und rechts zu verwenden.

Siehe auch gebundene und freie Variablen:
https://de.wikipedia.org/wiki/Freie_Variable_und_gebundene_Variable


Dir scheint aber die Wichtigkeit meiner Frage, ob du weist, wann
Funktionen oder Polynome gleich sind, nicht bewusst zu sein.

Du verwendest die Begriff "Funktion" aus der Analysis, bzw.
"Polynom" aus der Algebra. Beide mathematischen Teilbereiche
definieren hierfür die Gleichheit unterschiedlich.

Vielleicht eine leichtere Frage:

Wann sind zwei rationale Zahlen a/b und c/d mit den
ganzen Zahlen a, b, c und d gleich?

Kannst du wenigstens das hier hinschreiben, da du dich ja
zierst, die anderen Fragen zu beantworten.


So, dass müsste man aber auch in einem modernen Lehrbuch für Anfänger
finden, und diese Grndlagen solltest du schon verinnerlichen.

Aloha
IV
2018-04-17 18:58:21 UTC
Permalink
Post by Torn Rumero DeBrak
Ein Funktionsterm mit einem Bezeichner z
...
Rhabarber(f) := f*(a - 1/f)
Also, ich würde jetzt diese Begriffsbildungen aus der Informatik nicht mit
der in Analysis und Funktionentheorie gleichsetzen.
Post by Torn Rumero DeBrak
Dir scheint aber die Wichtigkeit meiner Frage, ob du weiszt, wann
Funktionen oder Polynome gleich sind, nicht bewusst zu sein.
Ja, die Wichtigkeit dieser Frage für meine Feststellung "f1 und f2
algebraisch unabhängig über dem Koeffizientenkörper \mathbb{C} => f1(z) und
f2(z) über Z1 \cap Z2 \neq \emptyset algebraisch unabhängig über dem
Koeffizientenkörper \mathbb{C}" ist mir nicht bewußt.
Post by Torn Rumero DeBrak
Du verwendest die Begriff "Funktion" aus der Analysis, bzw. "Polynom" aus
der Algebra. Beide mathematischen Teilbereiche definieren hierfür die
Gleichheit unterschiedlich.
Ich wüßte nicht, wo ich den Begriff "Polynom" verwendet hätte. Ein Polynom
alleine ist keine Polynomfunktion.
Post by Torn Rumero DeBrak
Wann sind zwei rationale Zahlen a/b und c/d mit den ganzen Zahlen a, b, c
und d gleich?
Ja, auch das ist eine leicht zu beantwortende Frage.
Post by Torn Rumero DeBrak
Kannst du wenigstens das hier hinschreiben, da du dich ja zierst, die
anderen Fragen zu beantworten.
Nö, kann ich nicht, da sich sonst gleich wieder H0Iger meldet.
Post by Torn Rumero DeBrak
So, dass müsste man aber auch in einem modernen Lehrbuch für Anfänger
finden, und diese Grndlagen solltest du schon verinnerlichen.
Ja. Und wozu brauchen wir das hier? Sag' es doch bitte.
H0Iger SchuIz
2018-04-18 07:14:20 UTC
Permalink
Post by IV
Post by Torn Rumero DeBrak
Wann sind zwei rationale Zahlen a/b und c/d mit den ganzen Zahlen a, b, c
und d gleich?
Ja, auch das ist eine leicht zu beantwortende Frage.
Post by Torn Rumero DeBrak
Kannst du wenigstens das hier hinschreiben, da du dich ja zierst, die
anderen Fragen zu beantworten.
Nö, kann ich nicht, da sich sonst gleich wieder H0Iger meldet.
Na, dann macht er das mal, um seiner besonderen Freude Ausdruck zu
geben, dass Jürgen einen Schuldigen dafür gefunden hat, dass er selbst
Beiträge zur Lösung seiner Probleme verweigert.

hs
Torn Rumero DeBrak
2018-04-18 07:59:51 UTC
Permalink
Post by IV
Post by Torn Rumero DeBrak
Ein Funktionsterm mit einem Bezeichner z
...
Rhabarber(f) := f*(a - 1/f)
Also, ich würde jetzt diese Begriffsbildungen aus der Informatik nicht
mit der in Analysis und Funktionentheorie gleichsetzen.
Post by Torn Rumero DeBrak
Dir scheint aber die Wichtigkeit meiner Frage, ob du weiszt, wann
Funktionen oder Polynome gleich sind, nicht bewusst zu sein.
Ja, die Wichtigkeit dieser Frage für meine Feststellung "f1 und f2
algebraisch unabhängig über dem Koeffizientenkörper \mathbb{C} => f1(z)
und f2(z) über Z1 \cap Z2 \neq \emptyset algebraisch unabhängig über dem
Koeffizientenkörper \mathbb{C}" ist mir nicht bewußt.
Post by Torn Rumero DeBrak
Du verwendest die Begriff "Funktion" aus der Analysis, bzw. "Polynom"
aus der Algebra. Beide mathematischen Teilbereiche definieren hierfür
die Gleichheit unterschiedlich.
Ich wüßte nicht, wo ich den Begriff "Polynom" verwendet hätte.
Implizit. "Algebraische (un)Abhängigkeit wird doch über die Existenz
oder Nicht-Existenz von Polynomen definiert. Oder sehe ich das falsch?
Post by IV
Ein
Polynom alleine ist keine Polynomfunktion.
Post by Torn Rumero DeBrak
Wann sind zwei rationale Zahlen a/b und c/d mit den ganzen Zahlen a,
b, c und d gleich?
Ja, auch das ist eine leicht zu beantwortende Frage.
Post by Torn Rumero DeBrak
Kannst du wenigstens das hier hinschreiben, da du dich ja zierst, die
anderen Fragen zu beantworten.
Nö, kann ich nicht, da sich sonst gleich wieder H0Iger meldet.
Post by Torn Rumero DeBrak
So, dass müsste man aber auch in einem modernen Lehrbuch für Anfänger
finden, und diese Grndlagen solltest du schon verinnerlichen.
Ja. Und wozu brauchen wir das hier? Sag' es doch bitte.
Du hast den Thread losgetreten. Warum braucht es das hier?

Aloha
IV
2018-04-18 16:02:37 UTC
Permalink
Post by IV
Post by Torn Rumero DeBrak
Post by IV
Post by Torn Rumero DeBrak
Dir scheint aber die Wichtigkeit meiner Frage, ob du weiszt, wann
Funktionen oder Polynome gleich sind, nicht bewusst zu sein.
Ja, die Wichtigkeit dieser Frage für meine Feststellung "f1 und f2
algebraisch unabhängig über dem Koeffizientenkörper \mathbb{C} => f1(z)
und f2(z) über Z1 \cap Z2 \neq \emptyset algebraisch unabhängig über
dem Koeffizientenkörper \mathbb{C}" ist mir nicht bewußt.
Du verwendest die Begriff "Funktion" aus der Analysis, bzw. "Polynom"
aus der Algebra. Beide mathematischen Teilbereiche definieren hierfür
die Gleichheit unterschiedlich.
Ich wüßte nicht, wo ich den Begriff "Polynom" verwendet hätte.
Implizit. "Algebraische (un)Abhängigkeit wird doch über die Existenz oder
Nicht-Existenz von Polynomen definiert. Oder sehe ich das falsch?
Es läuft auf Polynome hinaus, eigentlich heißen bei Ritt und mir zwei
Elemente aber algebraisch unabhängig, wenn zwischen ihnen keine algebraische
Funktion (oder algebraische Relation) besteht.

Ich sehe zwar nicht die Notwendigkeit, das hier zu bringen, aber damit wir
die Diskussion endlich abschließen können:
Zwei Funktionen f1: DB1 --> W1, z1 |--> f1(z1) und f2: DB2 --> W2, z2 |-->
f2(z2) sind genau dann gleich, wenn DB1 = DB2 und für jedes z \in DB1:
f1(z) = f2(z).
Zwei Polynome P1 und P2 sind genau dann gleich, wenn die Monome bzw.
Multinome von P1 mit denen von P2 übereinstimmen und umgekehrt.
Zwei rationale Zahlen a/b und c/d sind genau dann gleich, wenn a * d = b *
c.
Post by IV
Post by Torn Rumero DeBrak
So, dass müsste man aber auch in einem modernen Lehrbuch für Anfänger
finden, und diese Grndlagen solltest du schon verinnerlichen.
Ja. Und wozu brauchen wir das hier? Sag' es doch bitte.
Du hast den Thread losgetreten. Warum braucht es das hier?
Na, weil
- das Entscheidungsproblem der Existenz einer Umkehrfunktion einer
bijektiven Funktion in allgemeinen Funktionenklassen
und
- das Entscheidungsproblem der Auflösbarkeit von Gleichungen in allgemeinen
Funktionenklassen
nicht gelöst sind.
H0Iger SchuIz
2018-04-18 17:42:46 UTC
Permalink
Post by IV
Zwei Funktionen f1: DB1 --> W1, z1 |--> f1(z1) und f2: DB2 --> W2, z2 |-->
f1(z) = f2(z).
Die Wertebereiche muss man gar nicht beachten?

hs
IV
2018-04-19 18:32:15 UTC
Permalink
Post by H0Iger SchuIz
Post by IV
Zwei Funktionen f1: DB1 --> W1, z1 |--> f1(z1) und f2: DB2 --> W2, z2
|--> f2(z2) sind genau dann gleich, wenn DB1 = DB2 und für jedes z \in
DB1: f1(z) = f2(z).
Die Wertebereiche muss man gar nicht beachten?
Ihr seid doch die Mathematiker.
Ergeben sich die Wertebereiche denn nicht aus dem Gesagten?
Jens Kallup
2018-04-19 19:28:43 UTC
Permalink
Post by IV
Post by H0Iger SchuIz
Post by IV
Zwei Funktionen f1: DB1 --> W1, z1 |--> f1(z1) und f2: DB2 --> W2, z2
|--> f2(z2) sind genau dann gleich, wenn DB1 = DB2 und für jedes z
\in DB1: f1(z) = f2(z).
Die Wertebereiche muss man gar nicht beachten?
Ihr seid doch die Mathematiker.
Also ich hatte in meiner Lehre beigebracht bekommen, selbständig Dinge
oder Sachverhalte nachzuschlagen, zu korrigieren, oder zu ergänzen.
Dies trifft insbesondere auf Gesetzestexte zu - oder den AGB's.

Die können/werden sich mit laufender Zeit ändern.
Desweiteren haben meine Lehrer (denen Ich einen großen Dank hiermit
entsende an: Wolfgang, Holger, Arno, und und und ...) darauf geachtet,
selbständig zu argumentieren.

Sprich: "Eine Meinung nicht nachzuplappern, sondern zu Hinterfragen und
selbst eine Meinung zu bilden!".

Bei IV muss es immer wieder vorgefertigter Salat sein, was nicht gerade
die schöne Art ist.
Weiters: Was spricht dagegen, daß wir Dich überhaupt an Wissen rantasten
lassen bzw. müssen?

IV verlangt nur: Was der Mitnahmegesellschaft in Deutschland so richtig
Würze gibt - Also ich bin da kein Freund von.
Post by IV
Ergeben sich die Wertebereiche denn nicht aus dem Gesagten?
f1: DB1 --> W1, z1
f2: DB2 --> W2, z2

Damit meinst Du sicherlich, das DB1/DB2 der DefinitionsBereich1/-2 ist,
und mit W1/W2 := der Wertebereich1/-2 ist, sowie z1/z2 eine Zahl:
1 oder 2 ?

Es scheint, als würdest Du eine Menge gegen einen Wertebereich mit einer
bestimmten Zahl durchlaufen möchtest...??

Ich nehme an:

1. Bedingung für f1:
Du nimmst aus DB1 die Entität "Haus" und gehst nun die Straßen in einer
Stadt durch, bis Du die "Hausnummer" (als Beispiel: z1 := 42) gefunden
hast.

als 2. Bedingung für f2: möchtest Du offenbar das gleiche machen.
Jetzt aber mit einen anderen Datenmenge - ich nehme an:

DB1 := "Haus", W1 := 42
DB2 := "Bewohner", W2 := 3

Dies riecht stark nach Datenbank(durchläufe)...
Nun weiss ich nicht, ob man hier im Mathechannel Kenntnisse über
Datenbanken oder Objective Datenbanken geben soll.

Aber das weisst Du ja besser.

Gruß, Jens
Carlos Naplos
2018-04-20 06:08:19 UTC
Permalink
Post by IV
Post by H0Iger SchuIz
Post by IV
Zwei Funktionen f1: DB1 --> W1, z1 |--> f1(z1) und f2: DB2 --> W2, z2
|--> f2(z2) sind genau dann gleich, wenn DB1 = DB2 und für jedes z
\in DB1: f1(z) = f2(z).
Die Wertebereiche muss man gar nicht beachten?
Ihr seid doch die Mathematiker.
Ergeben sich die Wertebereiche denn nicht aus dem Gesagten?
Der Wertebereich ist ebenso wesentlich für eine Funktion wie der
Definitionsbereich.

Betrachte etwa die Mengen M und N, wobei N eine echte Teilmenge von M sei.
Betrachte ferner die Funktionen f: N -> N mit f(n) := n und g: N -> M
mit g(n) := n.

Dann ist f surjektiv, g jedoch nicht.
Ist das nicht ein wesentlicher Unterschied?
IV
2018-04-20 15:41:40 UTC
Permalink
Post by Carlos Naplos
Post by IV
Post by IV
Post by H0Iger SchuIz
Post by IV
Zwei Funktionen f1: DB1 --> W1, z1 |--> f1(z1) und f2: DB2 --> W2, z2
|--> f2(z2) sind genau dann gleich, wenn DB1 = DB2 und für jedes z \in
DB1: f1(z) = f2(z).
Die Wertebereiche muss man gar nicht beachten?
Ihr seid doch die Mathematiker.
Ergeben sich die Wertebereiche denn nicht aus dem Gesagten?
Der Wertebereich ist ebenso wesentlich für eine Funktion wie der
Definitionsbereich.
Betrachte etwa die Mengen M und N, wobei N eine echte Teilmenge von M sei.
Betrachte ferner die Funktionen f: N -> N mit f(n) := n und g: N -> M mit
g(n) := n.
Dann ist f surjektiv, g jedoch nicht.
Ist das nicht ein wesentlicher Unterschied?
Ah, ja, stimmt.
Weil es bei mir nur um surjektive Funktionen geht, hatte ich die Bild- und
Wertebereiche als aus der Definition oben herleitbar betrachtet.
(Besser ist, wenn Ihr solche mathematischen Dinge sagt, und nicht versucht
diese von mir zu erfragen. Ich bin kein Mathematik-Student und werde es auch
nie werden, ich bin in der Regel also unfähig, Eure Hausaufgaben- und
Testfragen wie ein solcher zu beantworten.)
H0Iger SchuIz
2018-04-20 15:47:35 UTC
Permalink
Post by IV
Ah, ja, stimmt.
Weil es bei mir nur um surjektive Funktionen geht,
Danach hatte Torn aber nicht gefragt. Aus Jürgens Antwort ging das auch
nicht hervor.
Post by IV
ich bin in der Regel also unfähig, Eure Hausaufgaben- und
Testfragen wie ein solcher zu beantworten.)
Eben. Auf die Gefahr hin, dass ich mich wiederhole, halte ich das
Unterfangen, Mathematik ohne Mathematikkenntnisse zu betreiben, für
aussichtslos.
IV
2018-04-20 19:18:47 UTC
Permalink
Post by H0Iger SchuIz
ich bin in der Regel also unfähig, Eure Hausaufgaben- und Testfragen wie
ein solcher zu beantworten.)
Eben. Auf die Gefahr hin, dass ich mich wiederhole, halte ich das
Unterfangen, Mathematik ohne Mathematikkenntnisse zu betreiben, für
aussichtslos.
Ich selbstverständlich auch. Ich kann ja nur Ideen und Anregungen geben.
Aber wenn sich nunmal kein Mathematiker findet.
H0Iger SchuIz
2018-04-20 06:23:13 UTC
Permalink
Post by IV
Ergeben sich die Wertebereiche denn nicht aus dem Gesagten?
Diese Frage stellt niemand, er _weiß_, wann Funktionen gleich sind.
Somit hat sich Torns nachbohren als erfolgreich bewiesen.
Torn Rumero DeBrak
2018-04-22 07:58:09 UTC
Permalink
Post by IV
Post by H0Iger SchuIz
Post by IV
Zwei Funktionen f1: DB1 --> W1, z1 |--> f1(z1) und f2: DB2 --> W2, z2
|--> f2(z2) sind genau dann gleich, wenn DB1 = DB2 und für jedes z
\in DB1: f1(z) = f2(z).
Die Wertebereiche muss man gar nicht beachten?
Ihr seid doch die Mathematiker.
Ergeben sich die Wertebereiche denn nicht aus dem Gesagten?
Nein, denn z.B.

die Funktion f1: IR^+ -> IR^+, x |-> x^2 ist bijektiv,

die Funktion f2: IR^+ -> IR, x |-> x^2 ist es nicht,

d.h. f1 und f2 sind unterschiedlich, obwohl sie denselben
Definitionsbereich haben und dieselbe definierende Zuordnungsvorschrift.

Deshalb wird eine Funktion auch aus Tripel definiert.

Aloha
Torn Rumero DeBrak
2018-04-22 07:46:39 UTC
Permalink
...
Post by IV
Post by Torn Rumero DeBrak
Du hast den Thread losgetreten. Warum braucht es das hier?
Na, weil
- das Entscheidungsproblem der Existenz einer Umkehrfunktion einer
bijektiven Funktion in allgemeinen Funktionenklassen
und
- das Entscheidungsproblem der Auflösbarkeit von Gleichungen in
allgemeinen Funktionenklassen
nicht gelöst sind.
Ja ist denn nicht schon gezeigt worden, dass die Entscheidungsprobleme
nicht endscheidbar sind (Turing usw.)?

Aloha
IV
2018-04-22 19:09:10 UTC
Permalink
Post by Torn Rumero DeBrak
Post by IV
Post by Torn Rumero DeBrak
Du hast den Thread losgetreten. Warum braucht es das hier?
Na, weil
- das Entscheidungsproblem der Existenz einer Umkehrfunktion einer
bijektiven Funktion in allgemeinen Funktionenklassen
und
- das Entscheidungsproblem der Auflösbarkeit von Gleichungen in
allgemeinen Funktionenklassen
nicht gelöst sind.
Ja ist denn nicht schon gezeigt worden, dass die Entscheidungsprobleme
nicht endscheidbar sind (Turing usw.)?
Hast Du Schlagworte oder Literaturstellen?
Ritt hat in [Ritt 1925] einen Struktursatz für Elementare Funktionen für das
erste Entscheidungsproblem gegeben.
Meiner >Einschätzung nach gibt es damit auch einen Struktursatz für das
zweite Entscheidungsproblem.
(Richardsons(?) Unmöglichkeitssatz hat die Computeralgebra für Jahrzehnte
ausgebremst, bis man festgestellt hat, daß dieser an sehr enge
Vorbedingungen geknüpft ist, die oftmals nicht zutreffen. Leider hat sich
diese Erkenntnis noch nicht bei allen rumgesprochen. Fragt nicht weiter,
denn mehr weiß ich darüber nicht.)
Torn Rumero DeBrak
2018-04-16 18:40:34 UTC
Permalink
Post by IV
Post by Torn Rumero DeBrak
Weisst du, wann zwei Funktionen gleich sind?
Ja.
https://de.wikibooks.org/wiki/Mathematik:_Analysis:_Grundlagen:_Funktionen
"Definition - Gleicheit von Funktionen"
Das sind NUR Links. Aber hast du den Sinn auch verstanden?
Deshalb solltest du besser mit EIGENEN Worten diese Begriffe
beschreiben.
Abschreiben ist kein Verstehen, und Links setzen erfordert noch nicht
einmal, den verlinkten Text zu lesen.
Post by IV
Post by Torn Rumero DeBrak
Weisst du, wann zwei Polynome mit Koeffizienten in einem Körper
gleich sind?
Ja.
https://www.google.de/search?q=%22Gleichheit+von+Polynomen%22&sourceid=ie7&rls=com.microsoft:de-DE:IE-Address&ie=&oe=&gfe_rd=cr&dcr=0&ei=0tnRWrrcCYiDX5jYg6gM
Und wozu brauchen wir das jetzt hier?
Du verwendest die Begriffen ziemlich inkonsequent. Deshalb
das Nachhaken, ob du die Gleichheit verstanden hast.
Post by IV
Wo ist ein Fehler in meiner
Argumentation/Notation? Sagt es doch bitte einfach.
Wenn du nicht sagst, was du meinst zu wissen, dann kann ich
natürlich auch nicht sagen, wo du deie Verständnisfehler begehst.

...
Post by IV
Mit welcher Notation kann man unterscheiden zwischen einem Term f(z) der
Variablen z und Funktionswerten f(z)?
siehe anderen Post.

Aloha
IV
2018-04-14 11:03:16 UTC
Permalink
Post by IV
Ich hatte auch eine Menge Fragen gestellt.
Wie kann man die algebraische Unabhängigkeit von Funktionen geeignet
definieren, ohne diese als Element eines (algebraisch abgeschlossenen)
Körpers zu definieren und ohne Termvereinfachungsalgorithmen, die ja nur
heuristisch und schlecht beweisbar sind, zu verwenden?
Joylyn Anderson
2018-04-21 05:22:50 UTC
Permalink
Post by IV
Hallo,
könnt Ihr mir bitte wieder helfen?
Es scheint der mathematische Satz unten zu gelten (siehe
https://mathoverflow.net/questions/296676/algebraic-independence-of-the-composition-of-functions
; Beweis folgt später an anderer Stelle).
Ist dann folgende Behauptung wahr?
Seien f1: Z1 Gebiet \subseteq \mathbb{C} \to \mathbb{C} und f2 \: Z2 Gebiet
\subseteq \mathbb{C} \to \mathbb{C} über dem Koeffizientenkörper \mathbb{C}
algebraisch voneinander unabhängige holomorphe Funktionen und A eine über
dem Koeffizientenkörper \mathbb{C} algebraische Funktion mit A: D \subseteq
\mathbb{C}^2, (z1,z2) \mapsto A(z1,z2).
Dann hat die Funktion F: Z Gebiet \subseteq \mathbb{C} \to \mathbb{C}, z
\mapsto A(f1(z),f2(z)) keine holomorphe Umkehrfunktion.
Denn dadurch, daß entsprechend dem Satz unten f1 \circ g und f2 \circ g auch
für die Umkehrfunktion g der Funktion F über dem Koeffizientenkörper
\mathbb{C} algebraisch unabhängig voneinander sind, läßt sich der Ausdruck
A(f1(g(z)),f2(g(z))) nicht zu A1(z), mit A1 eine über dem
Koeffizientenkörper \mathbb{C} algebraische Funktion mit A1: D{A1} \subseteq
\mathbb{C} \to \mathbb{C}, z \mapsto A1(z), vereinfachen, also auch nicht zu
z. (Wie kann man das beweisen?)
Seien f1, f2, g holomorphe Funktionen mit
f1: Z1 Gebiet \subseteq \mathbb{C} \to \mathbb{C},
f2: Z2 Gebiet \subseteq \mathbb{C} \to \mathbb{C},
g: Z3 Gebiet \subseteq \mathbb{C} \to \mathbb{C}, g nicht konstant,
und sei Z1 \cap Z2 \cap g(Z3) \neq \emptyset.
Wenn f1 und f2 über dem Koeffizientenkörper \mathbb{C} algebraisch
unabhängig voneinander sind, dann sind die Funktionen F1: Z \to \mathbb{C},
z \mapsto f1(g(z)) und F2: Z \to \mathbb{C}, z \mapsto f2(g(z)) über dem
Koeffizientenkörper \mathbb{C} algebraisch unabhängig voneinander.
Beide Sätze könnten bisher noch unbekannt zu sein. Sie könnten auch für die
Entscheidbarkeit der Auflösbarkeit von Gleichungen eine Rolle spielen. Es
wäre also lohnend, wenn Ihr hier Eure Erfahrung mit einbringen könntet.
Vielen Dank.
I have many hobbies. I like sport,swimming. I spend my life in moving.I love to cook .I love to sunbathe, walk, spend time outdoors. I hope that one day I will be able to do this with my beloved man .
Loading...